Diabetes & Metabolic Syndrome PQs

Réussis tes devoirs et examens dès maintenant avec Quizwiz!

B. The acetone on the woman's breath (which is produced by decarboxylation of acetoacetate; thus, E is incorrect) and the ketones in her urine indicate that she is in DKA. This is caused by low insulin levels, so her blood glucose levels are high because the glucose is not being taken up by the peripheral tissues (thus, A and C are incorrect). An insulin injection will reduce her blood glucose levels and decrease the release of fatty acids from adipose triglycerides. Consequently, ketone-body production will decrease. Glucagon injections would just exacerbate the woman's current condition (thus, D is incorrect).

A 20-year-old woman with diabetes mellitus was admitted to the hospital in a semiconscious state with fever, nausea, and vomiting. Her breath smelled of acetone. A urine sample was strongly positive for ketone bodies. Which one of the following statements about this woman is correct? A. A blood glucose test will probably show that her blood glucose level is well below 80 mg/dL. B. An injection of insulin will decrease her ketone-body production. C. She should be given a glucose infusion so she will regain consciousness. D. Glucagon should be administered to stimulate glycogenolysis and gluconeogenesis in the liver.

A: Pancreatic glucokinase. The boy has developed MODY (maturity onset diabetes of the young), and one variant of MODY is a mutated glucokinase (an inheritable disorder) such that the Km for glucose has increased, and insulin release only occurs when hyperglycemia is present. Both an increase in ATP and NADPH are required for the pancreatic β-cell to release insulin. When pancreatic glucokinase has an increased Km for glucose, ATP levels can only increase at greater than normal levels of glucose. Thus, moderate hyperglycemia is not sufficient to induce insulin release. As insulin release occurs from the pancreas, liver, muscle, or intestinal hexokinase will not affect the process. The pancreas does not express hexokinase, only glucokinase. MODY is a monogenetic autosomal dominant disease of insulin secretion. There are at least six amino acid substitutions known in a number of different proteins. MODY1 is a mutation in the transcription factor HNF4-α ∼ MODY2

A 28-year-old male develops diabetes, as noted by constant, mildly elevated hyperglycemia. His father had similar symptoms at the same age as did his paternal grandmother. This patient is not obese, does not have hypertension, does not have dyslipidemia, and does not have antibodies directed against islet cells. This alteration in glucose homeostasis may be due to a mutation in which of the following enzymes? (A) Pancreatic glucokinase (B) Pancreatic hexokinase (C) Liver glucokinase (D) Muscle hexokinase (E) Intestinal glucokinase

Gluconeogenesis is inhibited by ethanol This patient is exhibiting signs and clinical findings of acute alcohol ingestion. She is hypoglycemic and has lower than normal levels of insulin. Her serum toxicology profile indicates the presence of alcohol in the blood. The low serum glucose level in this patient may be attributable to the ability of ethanol to inhibit gluconeogenesis.

A 37-year-old woman collapses in front of a bar and is brought to the emergency department where the following data were obtained: Blood glucose, less than normal; insulin, lower than normal postabsorptive levels; epinephrine, higher than normal postabsorptive levels; C-peptide, normal; ethanol in blood, present. Which of the following descriptions of the patient is most consistent with the data provided? Presence of type II diabetes Presence of tumor of ß -cells of pancreas Gluconeogenesis is inhibited by ethanol Overdose of injected insulin Overdose of medication containing epinephrine

Correct answer = B. The oxidation of ethanol to acetate by dehydrogenases is accompanied by the reduction of nicotinamide adenine dinucleotide (NAD+) to NADH. The rise in the NADH/NAD+ ratio shifts pyruvate to lactate and oxaloacetate (OAA) to malate, decreasing the availability of substrates for gluconeogenesis and resulting in hypoglycemia. The rise in NADH also reduces the NAD+ needed for fatty acid (FA) oxidation. The decrease in OAA shunts any acetyl coenzyme A produced to ketogenesis. Note that the inhibition of FA degradation results in their reesterification into triacylglycerol that can result in fatty liver.

A 39-year-old woman is brought to the emergency room complaining of weakness and dizziness. She recalls getting up early that morning to do her weekly errands and had skipped breakfast. She drank a cup of coffee for lunch and had nothing to eat during the day. She met with friends at 8 p.m. and had a few drinks. As the evening progressed, she soon became weak and dizzy and was taken to the hospital. Laboratory tests revealed her blood glucose to be 45 mg/dl (normal = 70-99). She was given orange juice and immediately felt better. The biochemical basis of her alcohol-induced hypoglycemia is an increase in: A. fatty acid oxidation. B. the ratio of the reduced oxidized forms of nicotinamide adenine dinucleotide. C. oxaloacetate and pyruvate. D. use of acetyl coenzyme A in fatty acid synthesis.

A: Downregulation of the insulin receptors. The patient is exhibiting insulin resistance (persistently elevated blood glucose levels), most likely due to anti- body binding to the receptor, leading to downregulation of the receptor. Cells are thus kept in a persistently low insulin receptor state, blunting their ability to be stimu- lated by insulin. The lack of insulin signaling leads to elevated blood glucose levels. These alterations are not due to changes in glucagon secretion or activation of ade- nylate cyclase. Since muscle cells have downregulated insulin receptors, the muscle cells are not stimulated to remove glucose from the circulation. The antibodies also do not allow for upregulation of insulin recept

A 40-year-old man, with a BMI of 37, has a fasting blood glucose level of 165 mg/dL, a value that has been steadily increasing over the past 5 years. His HbA1c level is 8.9. Analysis of his blood, using Western blot technology, demonstrates the presence of antibodies against the insulin receptor. One consequence of these antibodies is which of the following? (A) Downregulation of the insulin receptors (B) Upregulation of the insulin receptors (C) Enhanced release of pancreatic glucagon (D) Enhanced glucose uptake into muscle cells (E) Activation of adenylate cyclase

A. Type 1 diabetes mellitus is caused by a lack of insulin (therefore, neither proinsulin nor C-peptide is produced as well), whereas type 2 diabetes mellitus is cellular resistance to secreted insulin (therefore, endogenous insulin and C-peptide are still produced in patients with type 2 diabetes). Measurement of an absolute insulin level would not be helpful because the patient is injecting insulin each day. However, if the patient is still producing insulin, he would also produce the C-peptide and would be classified as having type 2 diabetes. If C-peptide levels are not detected, the patient is classified as having type 1 diabetes. Individuals with type 1 diabetes can have islet cell antibodies in their blood, but not insulin antibodies. The presence of antibodies against insulin in the blood would lead to a reduced response to insulin, or a form of type 2 diabetes. The measurement of glucagon or proglucagon levels would not differentiate type 1 from type 2 diabetes because both glucagon and proglucagon would still be produced in individuals with diabetes. The levels of glucagon secreted in both types of diabetes is similar.

A 45-year-old patient was admitted to the hospital in a coma caused by severe hyperglycemia and was treated with insulin and fluids. He has been placed on long- and short-acting insulin injected daily to control his blood glucose levels. What test could be ordered at this point to determine if the patient has type 1 versus type 2 diabetes? A. C-peptide level B. Insulin level C. Insulin antibodies D. Proglucagon level E. Glucagon level

C. The boy has developed the signs and symptoms of untreated Type 1 diabetes. His pancreas is no longer producing insulin because of an autoimmune destruction of the β cells, so in response to elevated blood glucose levels insulin cannot be released. This leads to hyperglycemia, to the point that the kidney cannot reabsorb all of the glucose, and glucose is lost in the urine. The glucose in the urine causes an osmotic problem, and excess water is lost in the urine, leading to dehydration. This causes the frequent urination and thirst of the individual. The dehydration is leading to the lethargy and difficulty to awaken. Normal blood glucose levels should be 80 to 100 mg/dL in the fasting state. He has also developed a diabetic ketoacidosis, as the liver is producing ketone bodies in response to glucagon, but the brain is not utilizing them due to the high blood glucose levels. By giving the boy insulin, his blood glucose levels will stabilize as the muscle and fat cells begin to take glucose out of the circulation. Once blood glucose levels are below 300 mg/dL, glucose can be added to the iv bag. Rehydration is also occurring with normal saline as a part of the iv treatment.

A 6-year-old boy has become lethargic, was not eating well, but drinking copious amounts of water. He also urinated frequently, sometimes wetting the bed while sleeping. One morning he was difficult to rouse, and his parents took him to the emergency department, where a stat glucose showed 650 mg/dL, and a strip test for ketones was positive. The emergency-room physicians immediately placed the boy on an iv drip. For appropriate treatment, the iv drip should contain which one of the following at initial treatment? A. Glucose B. Glucagon C. Insulin D. Fatty acids E. Vitamins

C. The pH has dropped because of the high levels of ketone bodies in the blood, which occurs because the brain is using glucose as an energy source (since glucose levels are so high), and the brain is not using the ketone bodies. Lactate release by brain and muscle does not contribute to the acidosis, as the liver will utilize the lactate to generate glucose in the liver (since gluconeogenesis is stimulated due to the presence of glucagon, and the absence of insulin). Bicarbonate release could lead to an alkalosis, but not an acidosis. Under type-1-diabetic conditions, in the absence of insulin, the muscle cannot use glucose as an energy source, and it will use fatty acids for its energy needs (and ketone bodies to a limited extent).

A 6-year-old boy has become lethargic, was not eating well, but drinking copious amounts of water. He also urinated frequently, sometimes wetting the bed while sleeping. One morning he was difficult to rouse, and his parents took him to the emergency department, where a stat glucose showed 650 mg/dL, and a strip test for ketones was positive. The emergency-room physicians immediately placed the boy on an iv drip. The alteration in blood pH occurred due to which one of the following? A. Lactate release by the red blood cells B. Lactate release by the muscle C. Brain use of glucose as its fuel supply D. Muscle use of glucose as its fuel supply E. Bicarbonate release by the pancreas F. Bicarbonate release by the gall bladder

D. Insulin is required to stimulate glucose entry into muscle and fat cells. In the absence of insulin, a number of metabolic derangements occur. First, the liver believes the body is in fasting mode (since only glucagon is present), and the liver begins to export glucose from glycogenolysis and gluconeogenesis. Insulin is required for muscle and fat cells (but not brain cells) to take up glucose from the blood (through the increase in GLUT4 transporters in the membranes of those tissues). In the absence of insulin, and the inability of muscle and fat to use glucose in the circulation, glucose levels become elevated in the blood. The kidney actually allows glucose to enter the urine because of its high concentration, rather than keeping all of the glucose in the circulation.

A 6-year-old boy has become lethargic, was not eating well, but drinking copious amounts of water. He also urinated frequently, sometimes wetting the bed while sleeping. One morning he was difficult to rouse, and his parents took him to the emergency department, where a stat glucose showed 650 mg/dL, and a strip test for ketones was positive. The emergency-room physicians immediately placed the boy on an iv drip. The elevated glucose level exhibited by the boy is due, primarily, to which one of the following? A. Overeating of sweets B. Reduced number of glucose transporters in the brain C. Competition for glucose entry into the liver by fructose D. Reduced use of glucose by the muscle E. Reduced glucose filtration by the kidneys, resulting in more glucose in circulation

B. The boy is hyperglycemic, so the brain is getting plenty of fuel for its metabolic needs. Hyperglycemia can lead to lethargy as it will lead to severe dehydration, but this is not an answer choice. Fatty acids are actually increased in the blood due to the activation of hormone-sensitive lipase in the adipose cells. A hyperlipidemia would not lead to lethargy symptoms. The acidosis is what leads to the lethargy, as the acid−base balance of the blood and tissues is altered under these conditions. The acidosis results from excessive ketone bodies in circulation, since the brain is ignoring them in favor of the high blood glucose levels present.

A 6-year-old boy has become lethargic, was not eating well, but drinking copious amounts of water. He also urinated frequently, sometimes wetting the bed while sleeping. One morning he was difficult to rouse, and his parents took him to the emergency department, where a stat glucose showed 650 mg/dL, and a strip test for ketones was positive. The emergency-room physicians immediately placed the boy on an iv drip. The major reason for the boy's lethargy was which one of the following? A. An alkalosis B. An acidosis C. Hypoglycemia D. Hyperlipidemia E. Absence of fatty acids in the blood

Intravenous administration of insulin This patient has type I diabetes mellitus and would respond best to treatment with insulin. This condition is associated with destruction of the ß -cells of the pancreas which eliminates the production of insulin. Plasma insulin levels are low or absent. Treatment with glucagon will further increase blood glucose levels and worsen this situation. Type 1 Diabetes: - Destroyed β Cells - Age of Onset: Childhood or Puberty - Undernourished Pathophysiology: - Enlarged Peripancreatic Lymph Nodes - Cytokines (TNFα, IFNγ, IL-1) Diabetic Ketoacidosis: - ↓Insulin →↑FA Ox →↑↑↑Acetyl CoA →↑Ketones - ↑HSL (AT→FFAs) - Nausea & Vomiting - Abdominal Pain (Pancreatitis due to ↑↑↑TGs) - Kussmaul's breathing (↓HCO₃, Hyperkalemic Metabolic Acidosis) - Fruity Odor (Acetone) - Hyperglycemia - Dehydration (Glucosuria w/ Water Loss) - Hypotension & Coma - TX: Dextrose & Normal Saline (DNS) + Insulin Hyperglycemia: - ↑Glycogenolysis & ↑Gluconeogenesis - ↑A1c Hypertriglyceridemia: - ↑↑↑FFAs packed & disposed of as VLDL - ↓LPL - Glucagon Secretion Deficiency Treatment: - Insulin (Always Necessary)

A boy, age 12, with no previous prolonged history of disease, was admitted to the hospital complaining of increased frequency of urination, increased thirst, and increased appetite. On examination he showed signs of dehydration; the mucous membranes were dry, the skin was inelastic and wrinkled, and the eyeballs were sunken. Breathing was deep and rapid and the breath had the odor of acetone. A sample of blood provided the following data: Blood Glucose (mg%): 240 (Normal: 70-100) Blood pH: 7.1 (Normal: 7.3-7.5) Blood Ketone Bodies (mg%): 35 (Normal: 0-3) Urea (mmol/L: 8 (Normal: 2-7) Insulin (uU/mL): <1 (Normal: 20-30) Which of the following treatments would be most effective in rapidly normalizing the patient? Ingestion of a carbohydrate-free meal Ingestion of insulin plus a carbohydrate-containing meal Intravenous administration of glucagon Intravenous administration of epinephrine Intravenous administration of insulin

Circulating antibodies to pancreatic beta-cells When the diagnosis of type 1 diabetes is uncertain by clinical presentation, testing for circulating islet-cell antibodies is recommended. Type 1 Diabetes: - Destroyed β Cells - Age of Onset: Childhood or Puberty - Undernourished Pathophysiology: - Enlarged Peripancreatic Lymph Nodes - Cytokines (TNFα, IFNγ, IL-1) Diabetic Ketoacidosis: - ↓Insulin →↑FA Ox →↑↑↑Acetyl CoA →↑Ketones - ↑HSL (AT→FFAs) - Nausea & Vomiting - Abdominal Pain (Pancreatitis due to ↑↑↑TGs) - Kussmaul's breathing (↓HCO₃, Hyperkalemic Metabolic Acidosis) - Fruity Odor (Acetone) - Hyperglycemia - Dehydration (Glucosuria w/ Water Loss) - Hypotension & Coma - TX: Dextrose & Normal Saline (DNS) + Insulin Hyperglycemia: - ↑Glycogenolysis & ↑Gluconeogenesis - ↑A1c Hypertriglyceridemia: - ↑↑↑FFAs packed & disposed of as VLDL - ↓LPL - Glucagon Secretion Deficiency Treatment: - Insulin (Always Necessary)

A boy, age 12, with no previous prolonged history of disease, was admitted to the hospital complaining of increased frequency of urination, increased thirst, and increased appetite. On examination he showed signs of dehydration; the mucous membranes were dry, the skin was inelastic and wrinkled, and the eyeballs were sunken. Breathing was deep and rapid and the breath had the odor of acetone. A sample of blood provided the following data: Blood Glucose (mg%): 240 (Normal: 70-100) Blood pH: 7.1 (Normal: 7.3-7.5) Blood Ketone Bodies (mg%): 35 (Normal: 0-3) Urea (mmol/L: 8 (Normal: 2-7) Insulin (uU/mL): <1 (Normal: 20-30) Which of the following would be expected at the time of his admission to the hospital? Increased rate of glucose 6-phosphate oxidation in red blood cells Circulating antibodies to pancreatic beta-cells Insulin resistance Decreased levels of circulating free fatty acids Increased rate of restoration of depleted muscle glycogen following physical exercise

Elevated hemoglobin A1c Standard treatment typically consists of one or two daily injections of recombinant human insulin. Mean blood glucose levels obtained are typically in the 225-275 mg/dl range, with a hemoglobin A1C (HbA1C) level of 8-9% of the total hemoglobin. Type 1 Diabetes: - Destroyed β Cells - Age of Onset: Childhood or Puberty - Undernourished Pathophysiology: - Enlarged Peripancreatic Lymph Nodes - Cytokines (TNFα, IFNγ, IL-1) Diabetic Ketoacidosis: - ↓Insulin →↑FA Ox →↑↑↑Acetyl CoA →↑Ketones - ↑HSL (AT→FFAs) - Nausea & Vomiting - Abdominal Pain (Pancreatitis due to ↑↑↑TGs) - Kussmaul's breathing (↓HCO₃, Hyperkalemic Metabolic Acidosis) - Fruity Odor (Acetone) - Hyperglycemia - Dehydration (Glucosuria w/ Water Loss) - Hypotension & Coma - TX: Dextrose & Normal Saline (DNS) + Insulin Hyperglycemia: - ↑Glycogenolysis & ↑Gluconeogenesis - ↑A1c Hypertriglyceridemia: - ↑↑↑FFAs packed & disposed of as VLDL - ↓LPL - Glucagon Secretion Deficiency Treatment: - Insulin (Always Necessary)

A boy, age 12, with no previous prolonged history of disease, was admitted to the hospital complaining of increased frequency of urination, increased thirst, and increased appetite. On examination he showed signs of dehydration; the mucous membranes were dry, the skin was inelastic and wrinkled, and the eyeballs were sunken. Breathing was deep and rapid and the breath had the odor of acetone. A sample of blood provided the following data: Blood Glucose (mg%): 240 (Normal: 70-100) Blood pH: 7.1 (Normal: 7.3-7.5) Blood Ketone Bodies (mg%): 35 (Normal: 0-3) Urea (mmol/L: 8 (Normal: 2-7) Insulin (uU/mL): <1 (Normal: 20-30) Which of the following would be expected at the time of his admission to the hospital? Vascular disease Kidney disease Elevated hemoglobin A1c Hypoglycemia Neuropathy

Central nervous system (CNS) dysfunction Vascular disease is a long-term complication of diabetes. Glycogen depletion is not expected. Type 1 DM + Oversupply of Injected Insulin: - ↑↑↑Injected Insulin → ↑ Glucose to Tissue Cells → ↓Blood Glucose (Hypoglycemia) → ↑Epinephrine & ↑Glucagon - Injected insulin does NOT contain C-Peptide (↓blood levels) - ↑Frequency of Hypoglycemic Episodes, Coma, & Seizures - Patients with type 1 diabetes also develop a deficiency of glucagon secretion. These patients thus rely on epinephrine secretion to prevent severe hypoglycemia. ↑Insulin: - No Ketoacidosis & No Hyperlipidemia (↓Lipolysis) - ↑Glycogen Synthesis Type 1 Diabetes: - Destroyed β Cells - Age of Onset: Childhood or Puberty - Undernourished Pathophysiology: - Enlarged Peripancreatic Lymph Nodes - Cytokines (TNFα, IFNγ, IL-1) Diabetic Ketoacidosis: - ↓Insulin →↑FA Ox →↑↑↑Acetyl CoA →↑Ketones - ↑HSL (AT→FFAs) - Nausea & Vomiting - Abdominal Pain (Pancreatitis due to ↑↑↑TGs) - Kussmaul's breathing (↓HCO₃, Hyperkalemic Metabolic Acidosis) - Fruity Odor (Acetone) - Hyperglycemia - Dehydration (Glucosuria w/ Water Loss) - Hypotension & Coma - TX: Dextrose & Normal Saline (DNS) + Insulin Hyperglycemia: - ↑Glycogenolysis & ↑Gluconeogenesis - ↑A1c Hypertriglyceridemia: - ↑↑↑FFAs packed & disposed of as VLDL - ↓LPL - Glucagon Secretion Deficiency Treatment: - Insulin (Always Necessary)

A boy, age 8, with a confirmed history of insulin-dependent diabetes (type 1), was admitted to the hospital comatose and with a rapid heart rate. A sample of blood provided the following data: Blood Glucose (mg/100ml): 40 (Normal: 70-100) Blood pH: 7.3 (Normal: 7.3-7.5) Blood Ketone Bodies (mg/100ml): 2 (Normal: 0-3) Which of the following would be expected at the time of his admission to the hospital? Hyperlipidemia Severe ketoacidosis Central nervous system (CNS) dysfunction Vascular disease Glycogen depletion

Brain damage Type 1 DM + Oversupply of Injected Insulin: - ↑↑↑Injected Insulin → ↑ Glucose to Tissue Cells → ↓Blood Glucose (Hypoglycemia) → ↑Epinephrine & ↑Glucagon - Injected insulin does NOT contain C-Peptide (↓blood levels) - ↑Frequency of Hypoglycemic Episodes, Coma, & Seizures - Patients with type 1 diabetes also develop a deficiency of glucagon secretion. These patients thus rely on epinephrine secretion to prevent severe hypoglycemia. ↑Insulin: - No Ketoacidosis & No Hyperlipidemia (↓Lipolysis) - ↑Glycogen Synthesis Type 1 Diabetes: - Destroyed β Cells - Age of Onset: Childhood or Puberty - Undernourished Pathophysiology: - Enlarged Peripancreatic Lymph Nodes - Cytokines (TNFα, IFNγ, IL-1) Diabetic Ketoacidosis: - ↓Insulin →↑FA Ox →↑↑↑Acetyl CoA →↑Ketones - ↑HSL (AT→FFAs) - Nausea & Vomiting - Abdominal Pain (Pancreatitis due to ↑↑↑TGs) - Kussmaul's breathing (↓HCO₃, Hyperkalemic Metabolic Acidosis) - Fruity Odor (Acetone) - Hyperglycemia - Dehydration (Glucosuria w/ Water Loss) - Hypotension & Coma - TX: Dextrose & Normal Saline (DNS) + Insulin Hyperglycemia: - ↑Glycogenolysis & ↑Gluconeogenesis - ↑A1c Hypertriglyceridemia: - ↑↑↑FFAs packed & disposed of as VLDL - ↓LPL - Glucagon Secretion Deficiency Treatment: - Insulin (Always Necessary)

A boy, age 8, with a confirmed history of insulin-dependent diabetes (type 1), was admitted to the hospital complaining of dizziness and rapid heart rate. A sample of blood provided the following data: Blood Glucose (mg/100ml): 40 (Normal: 70-100) Blood pH: 7.3 (Normal: 7.3-7.5) Blood Ketone Bodies (mg/100ml): 2 (Normal: 0-3) Which of the following would be a cause for concern? Hyperglycemia Vascular disease Brain damage Severe ketoacidosis Glycogen depletion

Intravenous administration of glucagon Insulin would further lower blood glucose Ingested glucagon would be without effect due to hydrolysis of the peptide hormone in the GI tract A carbohydrate-free meal or an injection of lactate would be ineffective in reversing the observed hypoglycemia Type 1 DM + Oversupply of Injected Insulin: - ↑↑↑Injected Insulin → ↑ Glucose to Tissue Cells → ↓Blood Glucose (Hypoglycemia) → ↑Epinephrine & ↑Glucagon - Injected insulin does NOT contain C-Peptide (↓blood levels) - ↑Frequency of Hypoglycemic Episodes, Coma, & Seizures - Patients with type 1 diabetes also develop a deficiency of glucagon secretion. These patients thus rely on epinephrine secretion to prevent severe hypoglycemia. ↑Insulin: - No Ketoacidosis & No Hyperlipidemia (↓Lipolysis) - ↑Glycogen Synthesis Type 1 Diabetes: - Destroyed β Cells - Age of Onset: Childhood or Puberty - Undernourished Pathophysiology: - Enlarged Peripancreatic Lymph Nodes - Cytokines (TNFα, IFNγ, IL-1) Diabetic Ketoacidosis: - ↓Insulin →↑FA Ox →↑↑↑Acetyl CoA →↑Ketones - ↑HSL (AT→FFAs) - Nausea & Vomiting - Abdominal Pain (Pancreatitis due to ↑↑↑TGs) - Kussmaul's breathing (↓HCO₃, Hyperkalemic Metabolic Acidosis) - Fruity Odor (Acetone) - Hyperglycemia - Dehydration (Glucosuria w/ Water Loss) - Hypotension & Coma - TX: Dextrose & Normal Saline (DNS) + Insulin Hyperglycemia: - ↑Glycogenolysis & ↑Gluconeogenesis - ↑A1c Hypertriglyceridemia: - ↑↑↑FFAs packed & disposed of as VLDL - ↓LPL - Glucagon Secretion Deficiency Treatment: - Insulin (Always Necessary)

A boy, age 8, with a confirmed history of insulin-dependent diabetes (type 1), was admitted to the hospital complaining of dizziness and rapid heart rate. A sample of blood provided the following data: Blood Glucose (mg/100ml): 40 (Normal: 70-100) Blood pH: 7.3 (Normal: 7.3-7.5) Blood Ketone Bodies (mg/100ml): 2 (Normal: 0-3) Which of the following would be beneficial to the boy? Intravenous administration of glucagon Intravenous administration of a gluconeogenic precursor, such as lactate Ingestion of a carbohydrate-rich meal Ingestion of glucagon along with a meal containing complex carbohydrates Intravenous administration of insulin

Increased uptake of glucose by muscle Type 1 DM + Oversupply of Injected Insulin: - ↑↑↑Injected Insulin → ↑ Glucose to Tissue Cells → ↓Blood Glucose (Hypoglycemia) → ↑Epinephrine & ↑Glucagon - Injected insulin does NOT contain C-Peptide (↓blood levels) - ↑Frequency of Hypoglycemic Episodes, Coma, & Seizures - Patients with type 1 diabetes also develop a deficiency of glucagon secretion. These patients thus rely on epinephrine secretion to prevent severe hypoglycemia. ↑Insulin: - No Ketoacidosis & No Hyperlipidemia (↓Lipolysis) - ↑Glycogen Synthesis Type 1 Diabetes: - Destroyed β Cells - Age of Onset: Childhood or Puberty - Undernourished Pathophysiology: - Enlarged Peripancreatic Lymph Nodes - Cytokines (TNFα, IFNγ, IL-1) Diabetic Ketoacidosis: - ↓Insulin →↑FA Ox →↑↑↑Acetyl CoA →↑Ketones - ↑HSL (AT→FFAs) - Nausea & Vomiting - Abdominal Pain (Pancreatitis due to ↑↑↑TGs) - Kussmaul's breathing (↓HCO₃, Hyperkalemic Metabolic Acidosis) - Fruity Odor (Acetone) - Hyperglycemia - Dehydration (Glucosuria w/ Water Loss) - Hypotension & Coma - TX: Dextrose & Normal Saline (DNS) + Insulin Hyperglycemia: - ↑Glycogenolysis & ↑Gluconeogenesis - ↑A1c Hypertriglyceridemia: - ↑↑↑FFAs packed & disposed of as VLDL - ↓LPL - Glucagon Secretion Deficiency Treatment: - Insulin (Always Necessary)

A boy, age 8, with a confirmed history of insulin-dependent diabetes (type 1), was admitted to the hospital complaining of dizziness and rapid heart rate. A sample of blood provided the following data: Blood Glucose (mg/100ml): 40 (Normal: 70-100) Blood pH: 7.3 (Normal: 7.3-7.5) Blood Ketone Bodies (mg/100ml): 2 (Normal: 0-3) Which of the following would be expected at the time of his admission to the hospital? Increased lipolysis in adipose tissue Increased production of ketones by liver Increased breakdown of muscle glycogen Increased uptake of glucose by muscle Brain used ketone bodies as primary fuel

Increased levels of circulating insulin Type 1 DM + Oversupply of Injected Insulin: - ↑↑↑Injected Insulin → ↑ Glucose to Tissue Cells → ↓Blood Glucose (Hypoglycemia) → ↑Epinephrine & ↑Glucagon - Injected insulin does NOT contain C-Peptide (↓blood levels) - ↑Frequency of Hypoglycemic Episodes, Coma, & Seizures - Patients with type 1 diabetes also develop a deficiency of glucagon secretion. These patients thus rely on epinephrine secretion to prevent severe hypoglycemia. ↑Insulin: - No Ketoacidosis & No Hyperlipidemia (↓Lipolysis) - ↑Glycogen Synthesis Type 1 Diabetes: - Destroyed β Cells - Age of Onset: Childhood or Puberty - Undernourished Pathophysiology: - Enlarged Peripancreatic Lymph Nodes - Cytokines (TNFα, IFNγ, IL-1) Diabetic Ketoacidosis: - ↓Insulin →↑FA Ox →↑↑↑Acetyl CoA →↑Ketones - ↑HSL (AT→FFAs) - Nausea & Vomiting - Abdominal Pain (Pancreatitis due to ↑↑↑TGs) - Kussmaul's breathing (↓HCO₃, Hyperkalemic Metabolic Acidosis) - Fruity Odor (Acetone) - Hyperglycemia - Dehydration (Glucosuria w/ Water Loss) - Hypotension & Coma - TX: Dextrose & Normal Saline (DNS) + Insulin Hyperglycemia: - ↑Glycogenolysis & ↑Gluconeogenesis - ↑A1c Hypertriglyceridemia: - ↑↑↑FFAs packed & disposed of as VLDL - ↓LPL - Glucagon Secretion Deficiency Treatment: - Insulin (Always Necessary)

A boy, age 8, with a confirmed history of insulin-dependent diabetes (type 1), was admitted to the hospital complaining of dizziness and rapid heart rate. A sample of blood provided the following data: Blood Glucose (mg/100ml): 40 (Normal: 70-100) Blood pH: 7.3 (Normal: 7.3-7.5) Blood Ketone Bodies (mg/100ml): 2 (Normal: 0-3) Which of the following would be expected at the time of his admission to the hospital? Increased plasma levels of C-peptide Decreased entry of glucose into adipose cells Decreased levels of circulating hemoglobin A1c Increased levels of circulating insulin Decreased levels of circulating epinephrine and glucagon

Coma Type 1 DM + Oversupply of Injected Insulin: - ↑↑↑Injected Insulin → ↑ Glucose to Tissue Cells → ↓Blood Glucose (Hypoglycemia) → ↑Epinephrine & ↑Glucagon - Injected insulin does NOT contain C-Peptide (↓blood levels) - ↑Frequency of Hypoglycemic Episodes, Coma, & Seizures - Patients with type 1 diabetes also develop a deficiency of glucagon secretion. These patients thus rely on epinephrine secretion to prevent severe hypoglycemia. ↑Insulin: - No Ketoacidosis & No Hyperlipidemia (↓Lipolysis) - ↑Glycogen Synthesis Type 1 Diabetes: - Destroyed β Cells - Age of Onset: Childhood or Puberty - Undernourished Pathophysiology: - Enlarged Peripancreatic Lymph Nodes - Cytokines (TNFα, IFNγ, IL-1) Diabetic Ketoacidosis: - ↓Insulin →↑FA Ox →↑↑↑Acetyl CoA →↑Ketones - ↑HSL (AT→FFAs) - Nausea & Vomiting - Abdominal Pain (Pancreatitis due to ↑↑↑TGs) - Kussmaul's breathing (↓HCO₃, Hyperkalemic Metabolic Acidosis) - Fruity Odor (Acetone) - Hyperglycemia - Dehydration (Glucosuria w/ Water Loss) - Hypotension & Coma - TX: Dextrose & Normal Saline (DNS) + Insulin Hyperglycemia: - ↑Glycogenolysis & ↑Gluconeogenesis - ↑A1c Hypertriglyceridemia: - ↑↑↑FFAs packed & disposed of as VLDL - ↓LPL - Glucagon Secretion Deficiency Treatment: - Insulin (Always Necessary)

A boy, age 8, with a confirmed history of insulin-dependent diabetes (type 1), was brought to the emergency room complaining of dizziness and rapid heart rate. A sample of blood provided the following data: Blood Glucose (mg/100ml): 40 (Normal: 70-100) Blood pH: 7.3 (Normal: 7.3-7.5) Blood Ketone Bodies (mg/100ml): 2 (Normal: 0-3) Which of the following would be a cause for concern? Permanent loss of vision Coma Corneal clouding Liver disease Renal failure

Impairment in the stimulation of glucose transport into the peripheral tissues. The patient is exhibiting some of the symptoms of type II diabetes, a resistance to insulin action. Under these conditions the muscle and fat cells are not responding to the released insulin, so GLUT 4 transporters are not sent to the cell surface, and glucose levels remain elevated because the peripheral tissues are not taking up glucose at their normal rate. A defective glucokinase in the pancreas would result in insulin being released abnormally, most likely at higher than normal blood glucose levels, and that is not observed. A defective liver glucokinase would reduce glucose consumption of glucose, but would not affect the transport of glucose into the peripheral tissues. High glucose levels in the liver will stimulate glycogen synthesis, and eventually glycolysis in order to produce acetyl-CoA for fat synthesis.

A fifty year old overweight man visits his physician for a physical, and it is determined that his fasting blood glucose levels are elevated despite normal levels of insulin. An oral glucose tolerance test also demonstrates a normal rise in insulin levels after glucose is ingested, but an abnormal, slow drop in circulating blood glucose levels. These findings can be explained by which of the following? Impairment in the stimulation of glucose transport into the peripheral tissues. Allosteric inhibition of glycolysis by the circulating glucose. A defect in pancreatic glucokinase. Allosteric inhibition of glycogen synthesis by the circulating glucose. A defect in liver glucokinase.

a) 1) adipose tissue, lipolysis is faster; 2) muscle, glycogen synthesis is slower; 3) Liver, glycolysis, glycogen synthesis, and fatty acid synthesis are slower while gluconeogenesis is faster b) 1) hormone sensitive lipase is faster since it is activated by glucagon and opposed by insulin; 2) muscle, glycogen synthesis is slower since it requires activation by glycogen synthase via insulin signaling 3) Liver, glycolysis, glycogen synthesis, and fatty acid biosynthesis are slower since insulin signaling is required during the well-fed state while gluconeogenesis is faster since glucagon signaling is opposed by insulin signaling during the fasting state. If a patient does not comply with insulin therapy in this case due to forgetting it, then he/she may develop diabetes ketoacidosis and hyperglycemia, which require going to a clinic for treatment with fluid and electrolytes replacement due to dehydration and administering of short-acting insulin to gradually correct hyperglycemia without precipitating hypoglycemia. The adipose tissue would release high levels of fatty acids due to glucagon signaling and lack of insulin while the liver would be producing excess amount of keto bodies since fatty acid oxidation would be out of control. In addition, hyperglycemia would develop due to excess amount of glucose being released via gluconeogenesis and glycogen degradation. The muscle use ketone bodies and fatty acids via ß-oxidation for energy and for that insulin signaling is not necessary but glycogen synthesis will be slower since there is no insulin to signal that.

A man with insulin-dependent diabetes is brought to the emergency room in a near-comatose state. While vacationing in an isolated place, he lost his insulin medication and has not taken any insulin for two days. (a) For each tissue listed below, is each pathway faster, slower, or unchanged in this patient, compared with the normal level when he is getting appropriate amounts of insulin? (b) For each pathway, describe at least one control mechanism responsible for the change you predict. Tissue and Pathways 1. Adipose: lipolysis 2. Muscle: glycogen synthesis 3. Liver: glycolysis; gluconeogenesis; glycogen synthesis; fatty acid synthesis a) 1) adipose tissue, lipolysis is slower; 2) muscle, glycogen synthesis is slower; 3) Liver, glycolysis, glycogen synthesis, and fatty acid synthesis are slower while gluconeogenesis is faster b) 1) hormone sensitive lipase is slower since it is activated by glucagon and opposed by insulin; 2) muscle and glycogen synthesis are slower since it requires activation via insulin signaling 3) Liver, glycolysis, glycogen synthesis, and fatty acid biosynthesis are slower since insulin signaling is required during the well fed state while gluconeogenesis is faster since glucagon signaling is opposed by insulin signaling during the fasting state a) 1) adipose tissue, lipolysis is faster; 2) muscle, glycogen synthesis is slower; 3) Liver, glycolysis, glycogen synthesis, and fatty acid synthesis are slower while gluconeogenesis is faster b) 1) hormone sensitive lipase is faster since it is activated by glucagon and opposed by insulin; 2) muscle, glycogen synthesis is slower since it requires activation by glycogen synthase via insulin signaling 3) Liver, glycolysis, glycogen synthesis, and fatty acid biosynthesis are slower since insulin signaling is required during the well-fed state while gluconeogenesis is faster since glucagon signaling is opposed by insulin signaling during the fasting state. a) 1) adipose tissue, lipolysis is faster; 2) muscle and glycogen synthesis are slower; 3) Liver, glycolysis, glycogen synthesis, and fatty acid synthesis are faster while gluconeogenesis is slower b) 1) hormone sensitive lipase is faster since it is activated by glucagon and opposed by insulin; 2) muscle, glycogen synthesis is slower since it requires activation by glycogen synthase via insulin signaling 3) Liver, glycolysis, glycogen synthesis, and fatty acid biosynthesis are faster since insulin signaling is required during the well-fed state while gluconeogenesis is slower since glucagon signaling is opposed by insulin signaling during the fasting state a) 1) adipose tissue, lipolysis is faster; 2) muscle and glycogen synthesis are faster; 3) Liver, glycolysis, glycogen synthesis, and fatty acid synthesis are slower while gluconeogenesis is faster b) 1) hormone sensitive lipase is faster since it is activated by glucagon and opposed by insulin; 2) muscle, glycogen synthesis is faster since it requires activation by glycogen synthase via glucagon signaling 3) Liver, glycolysis, glycogen synthesis, and fatty acid biosynthesis are slower since insulin signaling during the well-fed state is required while gluconeogenesis is faster since glucagon signaling is opposed by insulin signaling during the fasting state

C. Insulin is required to stimulate glucose transport into muscle and fat cells but not into brain, liver, pancreas, or red blood cells. Thus, muscle would be feeling the effects of glucose deprivation and would be unable to replenish its own glycogen supplies as a result of its inability to extract blood glucose, even though blood glucose levels would be high.

A man with type 1 diabetes neglects to take his insulin injections while on a weekend vacation. Cells found within which tissue will be most greatly affected by this mistake? A. Brain B. Liver C. Muscle D. Red blood cells E. Pancreas

Although not measured, you would expect the levels of glucagon and epinephrine to be higher than normal Hypoglycemia will trigger increased release of glucagon and epinephrine in an effort to increase blood sugar. The patient is unlikely to be a diabetic due to normal Hb A1C. NADH produced by ethanol metabolism in the liver is responsive for lactate formation. Insulin is normal, so attempted suicide is an unlikely conclusion. The very mild ketoacidosis will not cause a large shift in brain metabolism.

A man, found unconscious on the street, is brought to the emergency room of the small-town hospital where you are finishing your postgraduate training. The man smelled of alcohol and an unfinished pint of vodka is found in his overcoat. It is early in the morning and you and one orderly are the total medical staff. The next morning the clinical laboratory provides the following data on the sample of blood initially obtained from the patient the previous night: Patient Normal Glucose 40 mg/100 mL 90 mg/100 mL Lactate 25 mg/100 mL 5 mg/100 mL beta-Hydroxybutyrate 20 mg/100 mL 5 mg/100 mL Hb A1C 5% Insulin 45 pmole/L 50 pmole/L pH 7.2 7.4 These data are most consistent with which ONE of the following statements? The patient should be put on suicide watch because a self-administered overdose of insulin is likely The patient is likely to be a type 1 diabetic Although not measured, you would expect the levels of glucagon and epinephrine to be higher than normal Blood lactate is elevated due to an increase in the intracellular NADH/NAD+ within the red blood cells, a tissue that depends on glycolysis for energy The patient's brain will be using roughly equal amounts of glucose and ketones as a fuel source

D. GIP and GLP-1 accentuate insulin release after a meal large enough to cause an increase in blood glucose concentration (so they should be increased as a treatment for diabetes). Both GIP and GLP-1 have a very short half-life owing to inactivation by DPP-4. Reducing the levels of DPP-4 would allow more GLP-1 and GIP to stimulate insulin release and lower postprandial blood glucose levels. Somatostatin and glucagon are counterregulatory hormones and would antagonize the effects of insulin.

A new patient to your practice has been diagnosed with type 2 diabetes. Your treatment plan includes prescribing a drug that would be beneficial in lowering postprandial serum glucose levels. A class of such a drug is which one of the following? A. Drugs that decrease levels of GIP B. Drugs that decrease levels of GLP-1 C. Drugs that increase levels of somatostatin D. Drugs that decrease levels of DPP-4 E. Drugs that increase the levels of glucagon

A. Sucrose and small amounts of glucose and fructose are the major natural sweeteners in fruit, honey, and vegetables. Lactose is the sugar found in milk and milk-derived products. Xylulose is a component of the pentose phosphate pathway, and its levels in fruits and vegetables are low.

A newly diagnosed patient with diabetes avoided table sugar because he knew he had "sugar diabetes," but he continued to consume fruits, fruit drinks, milk, honey, and vegetables, with the result being poor diabetic control. The diet the patient was following contained carbohydrate primarily in which form? Choose the one best answer. A. Sucrose B. Glucose C. Fructose D. Lactose E. Xylulose

D. The hyperglycemia in an untreated diabetic creates osmotic diuresis, which means that excessive water is lost through urination. This can lead to a contraction of blood volume, leading to low blood pressure and a rapid heartbeat. It also leads to dehydration. The rapid respirations results from acidosis-induced stimulation of the respiratory center of the brain in order to reduce the amount of acid in the blood. Ketone bodies have accumulated, leading to DKA (thus, B is incorrect). A patient in a hypoglycemic coma (which can be caused by excessive insulin administration) does not exhibit dehydration, low blood pressure, or rapid respirations; in fact, the patient will sweat profusely as a result of epinephrine release (thus, C and E are incorrect). Answer A is incorrect because lack of a pancreas would be fatal.

A patient arrives at the hospital in an ambulance. She is currently in a coma. Before lapsing into the coma, her symptoms included vomiting, dehydration, low blood pressure, and a rapid heartbeat. She also had relatively rapid respirations, resulting in more carbon dioxide being exhaled. These symptoms are consistent with which one of the following conditions? A. The patient lacks a pancreas. B. Ketoalkalosis C. Hypoglycemic coma D. DKA E. Insulin shock in a patient with diabetes

C: Elevated levels of sorbitol in the lens. Sorbitol synthesis from glucose in the polyol pathway occurs in the lens of the eye. Aldose reductase converts glucose to sorbitol which then accumulates in the lens. Sorbitol dehydrogenase can convert the sorbitol to fructose, which can also accumulate within the lens. In diabetes mellitus, fluctuating levels of glucose lead to fluctuating levels of sorbitol, which change the consis- tency of the lens and therefore the glasses prescription. Glucose and galactose by themselves do not directly affect the lens. Chronically, high glucose and sorbitol levels can increase cataract formation, but this patient is experiencing an acute problem. Hyperglycemia does not increase intraocular pressure; however, the conver- sion of glucose to sorbitol will.

A patient had new glasses prescribed by his optometrist. Less than a week later, his prescription was inadequate and he could not see well with his new glasses. His optometrist checked his vision twice more over the next week and the patient's prescription was different both times. His optometrist refers the patient to an ophthalmologist. What is the reason the patient is having such rapid changes in his glasses prescription? (A) Elevated levels of galactose in the lens (B) Elevated levels of glucose in the lens (C) Elevated levels of sorbitol in the lens (D) Cataract formation (E) Increased intraocular pressure from hyperglycemia

Reduced insulin release. Insulin is required for blood glucose to enter muscle and adipose tissues; the hyperglycemia observed two to four hours after the glucose challenge indicates that glucose is not entering cells at the appropriate rate, and is an indication that glucose intolerance is present. One possible reason for this is a reduced rate of release of insulin from the pancreas. If there were an increased insulin release, glucose levels should drop faster than the normal control, which was not observed. A decrease in glucagon release would enhance the uptake of glucose, as the insulin to glucagon ratio would be increased, but because this was not observed, that explanation cannot be the correct answer. Glucose metabolism in the liver does not directly affect the rate of glucose uptake by the peripheral tissues.

A patient has taken a glucose tolerance test (a large load of glucose is given orally to a fasting patient, and the rate of clearance of the glucose from the blood is measured as a function of time), and the results are shown below. Such a result could be due to which of the following? Reduced glucagon release. Reduced insulin release. Elevated glucose metabolism in the liver. Elevated insulin release. Reduced glucose metabolism in the liver.

Insulin C-peptide levels. Endogenous insulin is synthesized as preproinsulin. Once the pre sequence is removed, proinsulin is generated. Proinsulin is converted to mature insulin through removal of the C-peptide (the A and B peptides are linked by disulfide bonds, and form the mature, biologically active, insulin). C-peptide levels in the blood indicate that the insulin was synthesized by the ß -cells of the pancreas. Externally administered insulin lacks C-peptide, so if the insulin were injected, insulin levels would be high, but C-peptide levels would be very low. Measurement of the A or B chains of insulin would not be able to distinguish between exogenous and endogenous insulin. The measurement of blood glucose or glucagon levels would also not provide information as to the source of the elevated insulin in the blood.

A patient is found to be frequently hypoglycemic due to elevated insulin levels. One way to determine if the insulin is endogenous, or administered externally, is to measure which one of the following in the blood? Insulin A-peptide. Insulin C-peptide levels. Insulin B-peptide. Blood glucose levels. Blood glucagon levels.

B. The key to answering this question correctly relates to the absence of detectable C-peptide levels in the blood. An overproduction of insulin by the β-cells of the pancreas can lead to hypoglycemia severe enough to cause loss of consciousness, but because there was no detectable C-peptide in the blood, the loss of consciousness was most likely the result of the administration of exogenous insulin, which lacks the C-peptide (see Chapter 19). An overdose of glucagon (either through injection or from a glucagon-producing tumor), or epinephrine, would promote glucose release by the liver and not lead to hypoglycemia.

A patient is rushed to the emergency department after a fainting episode. Blood glucose levels were extremely low; insulin levels were normal, but there was no detectable C-peptide. The cause of the fainting episode may be which one of the following? A. An insulin-producing tumor B. An overdose of insulin C. A glucagon-producing tumor D. An overdose of glucagon E. An overdose of epinephrine

An overdose of insulin. The key to answering this question correctly relates to the absence of detectable C-peptide levels in the blood. An overproduction of insulin by the ß cells can lead to hypoglycemia severe enough to cause loss of consciousness, but since there was no detectable C-peptide in the blood, the loss of consciousness was most likely the result of the administration of exogenous insulin which lacks the C-peptide (see chapter 26). An overdose of glucagon, or epinephrine, would promote glucose release by the liver, and not lead to hypoglycemia. Type 1 DM + Oversupply of Injected Insulin: - ↑↑↑Injected Insulin → ↑ Glucose to Tissue Cells → ↓Blood Glucose (Hypoglycemia) → ↑Epinephrine & ↑Glucagon - Injected insulin does NOT contain C-Peptide (↓blood levels) - ↑Frequency of Hypoglycemic Episodes, Coma, & Seizures - Patients with type 1 diabetes also develop a deficiency of glucagon secretion. These patients thus rely on epinephrine secretion to prevent severe hypoglycemia. ↑Insulin: - No Ketoacidosis & No Hyperlipidemia (↓Lipolysis) - ↑Glycogen Synthesis Type 1 Diabetes: - Destroyed β Cells - Age of Onset: Childhood or Puberty - Undernourished Pathophysiology: - Enlarged Peripancreatic Lymph Nodes - Cytokines (TNFα, IFNγ, IL-1) Diabetic Ketoacidosis: - ↓Insulin →↑FA Ox →↑↑↑Acetyl CoA →↑Ketones - ↑HSL (AT→FFAs) - Nausea & Vomiting - Abdominal Pain (Pancreatitis due to ↑↑↑TGs) - Kussmaul's breathing (↓HCO₃, Hyperkalemic Metabolic Acidosis) - Fruity Odor (Acetone) - Hyperglycemia - Dehydration (Glucosuria w/ Water Loss) - Hypotension & Coma - TX: Dextrose & Normal Saline (DNS) + Insulin Hyperglycemia: - ↑Glycogenolysis & ↑Gluconeogenesis - ↑A1c Hypertriglyceridemia: - ↑↑↑FFAs packed & disposed of as VLDL - ↓LPL - Glucagon Secretion Deficiency Treatment: - Insulin (Always Necessary)

A patient is rushed to the emergency room after a fainting episode. Blood glucose levels were extremely low; insulin levels were normal, but there was no detectable C-peptide. The cause of the fainting episode may be due to which of the following? A glucagon producing tumor. An overdose of insulin. An insulin producing tumor. An overdose of epinephrine. An overdose of glucagon.

B: The baby's relative hyperinsulinemia. During pregnancy, the fetus is oversupplied with glu- cose from the mother causing the fetal pancreas to overproduce insulin. At delivery, the glucose supply from the mother is suddenly terminated and the rela- tive hyperinsulinemia of the baby causes hypoglycemia until the baby's body can adjust to this new environ- ment by decreasing insulin release and increasing glu- cose release. Hypoglycemia in the first few hours of the newborn's life is a common complication of gestational diabetes. Newborn hyperglycemia would not give a heel stick of 30 mg/dL of glucose. The placenta does not make insulin and the insulin molecule cannot cross the placenta, so the mother's relative hyperinsulinemia is not the cause of this problem. While the mother's hyper- glycemia has led to the baby's relative hyperinsulinemia, the mother's blood glucose levels do not cause the drop in the baby's blood glucose levels after birth.

A patient who had gestational diabetes has just delivered a 10 lb baby. The baby appears "jittery" and a heel stick glucose is 30 mg/dL. Which of the following mechanisms is the explanation for the newborn's blood glucose reading? (A) The mother's relative hyperinsulinemia (B) The baby's relative hyperinsulinemia (C) The mother's hyperglycemia (D) The baby's hyperglycemia (E) Placental insulin production

C. Acetoacetate and β-hydroxybutyrate are the ketone bodies produced by the liver. Acetoacetate can be converted to acetone and CO2. Because acetone is volatile, it is expired by the lungs. In ketoacidosis, increased production of acetone gives the classic odor to the breath.

A patient with diabetes in ketoacidosis has a specific odor to the breath. Which one of the following compounds is responsible for this odor? A. Acetoacetate B. β-Hydroxybutyrate C. Acetone D. Acetyl-CoA E. CO2

C. Insulin is required for glucose transport into both skeletal muscle and adipose tissue but is not required for glucose uptake into the red blood cells or brain (nervous tissue). The tissues that cannot transport glucose will be unable to metabolize it under these conditions. The inability of skeletal muscle and adipose tissue to take up glucose in the absence of insulin contributes to the high blood glucose levels seen in people with type 1 diabetes who have neglected to take their insulin.

A patient with type 1 diabetes has neglected to take his insulin before eating a carbohydrate-rich meal. Which of the following tissues will metabolize glucose under these conditions? Choose the ONE best answer.

D. When the patient took the insulin, the hormone stimulated glucose transport into the muscle and fat cells. This had the effect of lowering blood glucose levels, and, by not eating, the patient became severely hypoglycemic to the point that the blood glucose levels were below the Km for the glucose transporters for the nervous system. The administration of epinephrine will stimulate the liver to release glucose, via glycogenolysis and gluconeogenesis, and will raise blood glucose levels sufficiently to overcome the insulin-induced hypoglycemia. The addition of insulin would only exacerbate the problem. Addition of triglycerides will not aid the nervous system because the fatty acids cannot cross the blood-brain barrier. Normal saline will not add nutrients for the nervous system. Short-chain fatty acids also cannot enter the nervous system.

A patient with type 1 diabetes mellitus takes an insulin injection before eating dinner but then gets distracted and does not eat. Approximately 3 hours later, the patient becomes shaky, sweaty, and confused. If the patient had fallen asleep before recognizing the symptoms, the patient could lose consciousness while sleeping. If that were to occur and paramedics were called to help the patient, the administration of which one of the following would help to reverse this effect? A. Insulin B. Normal saline C. Triglycerides D. Epinephrine E. Short-chain fatty acids

D. Once insulin is injected, glucose transport into the peripheral tissues will be enhanced. If the patient does not eat, the normal fasting level of glucose will drop even further resulting from the injection of insulin, which increases the movement of glucose into muscle and fat cells. The patient becomes hypoglycemic, as a result of which epinephrine is released from the adrenal medulla. This, in turn, leads to the signs and symptoms associated with high levels of epinephrine in the blood. Answers A and B are incorrect because as glucose levels drop, glucagon will be released from the pancreas to raise blood glucose levels, which would alleviate the symptoms. Answer E is incorrect because ketone body production does not produce hypoglycemic symptoms, nor would ketone bodies be significantly elevated only a few hours after the insulin shock the patient is experiencing.

A patient with type 1 diabetes mellitus takes an insulin injection before eating dinner but then gets distracted and does not eat. Approximately 3 hours later, the patient becomes shaky, sweaty, and confused. These symptoms have occurred because of which one of the following? A. Increased glucagon release from the pancreas B. Decreased glucagon release from the pancreas C. High blood glucose levels D. Low blood glucose levels E. Elevated ketone-body levels

B. Insulin stimulates the transport of glucose into adipose and muscle cells by promoting the recruitment of GLUT 4 glucose transporters to the cell membrane. Liver, brain, intestine, and red blood cells have different types of glucose transporters that are not significantly affected by insulin.

A patient with type 1 diabetes, who has forgotten to take insulin before a meal, will have difficulty assimilating blood glucose into which one of the following tissues? A. Brain B. Adipose C. Red blood cell D. Liver E. Intestine

The answer is A. Bread and potatoes are high in starch/carbohydrates. A high-carbohydrate meal stimulates the release of insulin from the β-cells of the pancreas to help use and store this available glucose. Glucose and insulin suppress the release of glucagon from the α-cells of the pancreas. Glucagon helps to generate glucose from endogenous stores, which is unnecessary with such high levels of glucose from dietary sources. Salivary and pancreatic amylase are both required in the breakdown of dietary starches to glucose. Lactase is required to convert lactose to glucose and galactose (from the milk sugar in the meal).

A patient with type 2 diabetes mellitus takes his insulin shot and then 15 minutes later ingests a meal of bread, potatoes, and milk. Which ONE of the following proteins is suppressed during the absorption of this meal? A. Glucagon B. Insulin C. Salivary amylase D. Pancreatic amylase E. Lactase

B. Insulin is required for muscle and fat cells to synthesize and release LPL to bind to the capillary walls of the capillaries supplying these tissues with blood. In the absence of insulin, LPL levels would be reduced, and there would be a reduced rate of triglyceride removal from both chylomicrons and VLDL particles. Chylomicron synthesis would be normal on a normal diet. The problem is that the lipid is not leaving the chylomicron as rapidly as it normally would because of the reduced levels of LPL on the capillary walls. VLDL synthesis would actually increase, owing to fatty acid synthesis by the liver (because of the high levels of glucose in the blood), and because of excess fatty acids reaching the liver owing to increased lipolysis of triacylglycerol in the fat cell, as induced by the absence of insulin.

A person with type 1 diabetes who has neglected to take insulin for 2 days, yet who ate normally, would exhibit elevated levels of circulating triglycerides because of which one of the following? A. Reduced synthesis of apolipoprotein CII B. Reduced release of LPL from muscle and fat cells C. Increased synthesis of LDL D. Reduced synthesis of chylomicrons E. Reduced synthesis of VLDL

A. The liver will produce ketone bodies when fatty acid oxidation is increased, which occurs when glucagon is the predominant hormone (glucagon leads to fatty acid release from the fat cells for oxidation in the liver and muscle). This would be the case in an individual who cannot produce insulin and is not taking insulin injections. However, in this situation, the ketone bodies are not being used by the nervous system (brain) because of the high levels of glucose in the circulation. This leads to severely elevated ketone levels because of non-use. The glucose is high because, in the absence of insulin, muscle and fat cells are not using the glucose in circulation as an energy source. Recall that although the liver produces ketone bodies, it lacks a necessary enzyme to use ketone bodies as an energy source. There is no relation between BUN levels and the rate of ketone body production. The muscle reduces its use of ketone bodies under these conditions but not its use of fatty acids.

A physician is treating a type 1 diabetic who has neglected to take her insulin for 5 days. The patient demonstrates elevated blood glucose and ketone body levels. Ketone bodies are elevated because of which one of the following? A. Elevated glucose levels B. Reduced BUN C. Decreased fatty acid release from the adipocyte D. Inhibition of liver oxidation of ketone bodies E. Reduced muscle use of fatty acids

The generation of reactive oxygen intermediates. Elevation in intracellular glucose levels can cause an increased flux through the enzyme aldose reductase. Aldose reductase uses NADPH to reduce glucose to sorbitol which is then oxidized to fructose. The increased level of intracellular sorbitol may cause osmotic damage to tissues. Hyperglycemia is also associated with the nonenzymatic glycation of proteins yielding products of varying chemical structure known as advanced glycosylation endproducts (AGEs). Formation of AGEs may damage cells by impairing the function of a wide range of proteins such as collagen. Hyperglycemia can increase oxidative stress through both enzymatic and nonenzymatic processes. Byproducts of mitochondrial oxidative phosphorylation include free radicals such as superoxide anion. Their generation is increased by high glucose levels. These free oxygen radicals can damage cellular proteins and promote leukocyte adhesion to the endothelium while inhibiting its barrier function. Protein kinase A is activated when glucose levels are low, via glucagon release from the pancreas. Chronic hyperglycemia will not affect this, nor will elevated PKA activity lead to vascular complications. In addition, the mesangial matrix has not been shown to express angiotensin II.

A postulated mechanism by which chronic hyperglycemia contributes to the development of the microvascular complications of chronic diabetes mellitus is which of the following? Decreased flux through the aldose reductase pathway. The decreased production of advanced glycosylation endproducts. The generation of reactive oxygen intermediates. Excessive activation of protein kinase A (PKA). Increased production of angiotensin II within the mesangial matrix.

B: The anabolic effects of insulin. In pregnancy, the placenta preferentially shunts glucose to the developing fetus. This, along with placental hor- mones, causes a functional "insulin resistance" in the mother. Because of the higher glucose level in the fetus, the fetal pancreas produces more insulin. Insulin is the major anabolic hormone of the body stimulating glu- cose uptake into the cells and stimulating extra growth. Glucagon, growth hormone, epinephrine, corticoster- oids, and thyroid hormone are all catabolic hormones that counter insulin and stimulate glucose release from the cells to increase blood glucose. Glucose itself is nei- ther anabolic nor catabolic. Large for gestational age babies, dehydrated babies (from the osmotic diuresis of hyperglycemia), and a fivefold increase in stillborn rates are all complications of uncontrolled gestational diabetes.

A pregnant patient has developed gestational diabetes. One of the consequences of gestational diabetes is fetal macrosomia. Which of the following is the mechanism that causes these large for gestational age babies? (A) The anabolic effects of glucose (B) The anabolic effects of insulin (C) The anabolic effects of glucagon (D) The anabolic effects of growth hormone (E) The anabolic effects of thyroid hormone

B. Blood glucose levels peak approximately 1 hour after eating and return to the fasting range by about 2 hours. If the blood glucose levels remain elevated for an extended period of time, it is an indication of impaired glucose transport (insulin stimulates glucose transport into muscle and adipose tissue). If the blood glucose levels are <140 mg/dL at 2 hours after the test, the result is considered normal. If the levels are between 140 and 200 mg/dL, the patient is considered to have "impaired glucose tolerance." If the levels are >200 mg/dL after 2 hours, a diagnosis of diabetes is confirmed.

A pregnant woman is having an oral glucose tolerance test done to diagnose gestational diabetes. The test consists of ingesting a concentrated solution of glucose (75 g of glucose) and then having her blood glucose levels measured at various times after ingesting the sugar. Her test results come back normal. At what time after her oral sugar solution in consumed will she have the highest blood glucose level? A. Immediately B. 1 hour C. 2 hours D. 3 hours E. 4 hours

Defective GLUT 2 molecules. Insulin release from the pancreas is initiated by an increase in the [ATP]/[ADP] ratio in the ß cells; thus any type of mutation that requires a higher glucose level to initiate insulin release will disrupt insulin release. GLUT 2 transporters carry glucose from the blood into ß cells. If these transporters are defective, higher than normal levels of glucose are required to generate the appropriate level of glucose metabolism in the ß cells to initiate insulin release.

A twelve-year-old boy is found to be hyperglycemic upon a routine visit to his physician's office. A glucose tolerance test indicated that the child maintained his resting blood glucose levels at a level higher than normal. Tests for insulin in the blood indicated that the child could synthesize and release insulin given a high enough glucose challenge. PCR analysis of DNA obtained from blood cells indicated that the pancreatic glucokinase gene did not contain any of the known mutations for maturity onset diabetes of the young (MODY). Insulin binding to fibroblasts cultured from the child indicated no defect. One possible mutation that could account for these findings is which of the following? Defective GLUT 2 molecules. Defective GLUT 5 molecules. Defective GLUT 4 molecules. Defective muscle protein kinase A. Defective liver protein kinase A.

Epinephrine. When the patient took the insulin the hormone stimulated glucose transport into the muscle and fat cells. This had the effect of lowering blood glucose levels, and, by not eating, the patient became severely hypoglycemic, to the point that the blood glucose levels were below the Km for the glucose transporters for the nervous system. The administration of epinephrine will stimulate the liver to release glucose, via glycogenolysis and gluconeogenesis, and will raise blood glucose levels sufficiently to overcome the insulin-induced hypoglycemia. The addition of insulin would only exacerbate the problem. Addition of triglycerides will not aid the nervous system, as the fatty acids cannot cross the blood brain barrier. Normal saline will not add nutrients for the nervous system. Short-chain fatty acids also cannot enter the nervous system.

A type 1 diabetic injected insulin before dinner, but was distracted and did not eat after the injection. During the night the patient became lethargic, and lost consciousness. Administration of which one of the following can help to reverse this effect? Triglycerides. Short-chain fatty acids. Insulin. Epinephrine. Normal saline.

D: Reduced secretion of LPL. Insulin release stimulates the secretion of lipoprotein lipase (LPL) from fat and muscle cells such that the capillaries infiltrating these tissues have the lipase bound to extra- cellular matrix material. Then, as the triglyceride-rich particles move through the tissues, they bind to LPL via apolipoprotein CII, and the triglyceride is digested and the fatty acids used by the tissues. In the absence of insulin, LPL levels are low, and the particles have a longer half-life in circulation due to the reduced rate of digestion, which contributes to hypertriglyceridemia. If there were reduced synthesis of VLDL, triglycerides in the circulation would be reduced, not increased. Insulin does not alter the rate of apolipoprotein CII production. The release of insulin decreases fatty acid oxidation (promoting fatty acid synthesis), but if increased fatty acid oxidation did occur, then triglycerides would not accumulate in the circulation. Insulin also does not alter the synthesis of apolipoprotein B100 in the liver, which is required for VLDL synthesis.

A type 1 diabetic who has neglected to take his insulin for a few days displays both hyperglycemia and hypertriglyceridemia. The hypertriglyceridemia is due, in part, to which of the following? (A) Reduced synthesis of VLDL (B) Reduced production of apolipoprotein CII (C) Increased fatty acid oxidation (D) Reduced secretion of LPL (E) Increased synthesis of B100

B: Enhance fatty acid oxidation. Met- formin, through its activation of the AMP-activated pro- tein kinase, will stimulate glucose entry into the muscle (thus, answer choice A is incorrect) and also increase fatty acid oxidation. The AMP-activated protein kinase will phosphorylate and inhibit acetyl-CoA carboxylase and will phosphorylate and activate malonyl-CoA decar- boxylase. Thus, malonyl-CoA levels drop, leading to enhanced entry of fatty acids into the mitochondria, and an increase in fatty acid oxidation. This occurs as the malonyl-CoA inhibition of carnitine palmityl transferase 1 is now lifted due to the reduction of malonyl-CoA levels (see the figure below). Metformin does not stimu- late glucose release from the muscle, as the muscle lacks glucose-6-phosphatase activity. Metformin also reduces gluconeogenesis in the liver at a transcriptional level

A type 2 diabetic has been taking metformin to help regulate blood glucose levels. What effect will metformin also exert within the muscle? (A) Reduce glucose uptake from the circulation (B) Enhance fatty acid oxidation (C) Reduce fatty acid oxidation (D) Stimulate glucose release (E) Enhance gluconeogenesis

Increased Glucose Transport by the Muscle [Yes] VLDL Synthesis by the Liver [Yes] Triglyceride Storage in Adipose Tissue [Yes] Triglyceride Synthesis in Fat Cells [Yes] Fatty Acid Synthesis in Fat Cells [No] Glycogen Synthesis in the Liver [Yes] In the fed state, and in the presence of insulin, glucose transport into both the adipocyte and muscle cell will be increased. Insulin will also stimulate the liver to synthesize both glycogen and fatty acids, which leads to enhanced triglyceride synthesis and VLDL production to deliver the fatty acids to other tissues of the body. Insulin will stimulate glucose uptake in fat cells but does not stimulate fatty acid synthesis in the fat cells (that is unique to the liver) but will lead to enhanced triglyceride synthesis in the fat cells.

A type 2 diabetic, whose diabetes is well controlled (as determined by his hemoglobin A1c levels), has taken his insulin and 15 minutes later eats his evening meal. Which of the following changes will occur as he digests his meal? Increased Glucose Transport by the Muscle [Yes or No] VLDL Synthesis by the Liver [Yes or No] Triglyceride Storage in Adipose Tissue [Yes or No] Triglyceride Synthesis in Fat Cells [Yes or No] Fatty Acid Synthesis in Fat Cells [Yes or No] Glycogen Synthesis in the Liver [Yes or No]

Gluconeogenesis is inhibited by ethanol The consumption and subsequent metabolism of ethanol inhibits gluconeogenesis, leading to hypoglycemia in individuals with depleted stores of liver glycogen. Alcohol consumption can also increase the risk for hypoglycemia in patients using insulin. Chronic alcohol consumption can cause liver disease.

A woman collapses in front of a bar and is rushed to the emergency room where the following data were obtained: blood glucose: less than normal; insulin: lower than normal postabsorptive levels; epinephrine: higher than normal postabsorptive levels; C-peptide: normal; ethanol in blood: present. Which ONE of the following descriptions of the patient is most consistent with the data provided? Presence of tumor of beta-cells of pancreas Overdose of medication containing epinephrine Gluconeogenesis is inhibited by ethanol Overdose of injected insulin Presence of type II diabetes

Increased levels of circulating epinephrine and glucagon Type 1 DM + Oversupply of Injected Insulin: - ↑↑↑Injected Insulin → ↑ Glucose to Tissue Cells → ↓Blood Glucose (Hypoglycemia) → ↑Epinephrine & ↑Glucagon - Injected insulin does NOT contain C-Peptide (↓blood levels) - ↑Frequency of Hypoglycemic Episodes, Coma, & Seizures - Patients with type 1 diabetes also develop a deficiency of glucagon secretion. These patients thus rely on epinephrine secretion to prevent severe hypoglycemia. ↑Insulin: - No Ketoacidosis & No Hyperlipidemia (↓Lipolysis) - ↑Glycogen Synthesis Type 1 Diabetes: - Destroyed β Cells - Age of Onset: Childhood or Puberty - Undernourished Pathophysiology: - Enlarged Peripancreatic Lymph Nodes - Cytokines (TNFα, IFNγ, IL-1) Diabetic Ketoacidosis: - ↓Insulin →↑FA Ox →↑↑↑Acetyl CoA →↑Ketones - ↑HSL (AT→FFAs) - Nausea & Vomiting - Abdominal Pain (Pancreatitis due to ↑↑↑TGs) - Kussmaul's breathing (↓HCO₃, Hyperkalemic Metabolic Acidosis) - Fruity Odor (Acetone) - Hyperglycemia - Dehydration (Glucosuria w/ Water Loss) - Hypotension & Coma - TX: Dextrose & Normal Saline (DNS) + Insulin Hyperglycemia: - ↑Glycogenolysis & ↑Gluconeogenesis - ↑A1c Hypertriglyceridemia: - ↑↑↑FFAs packed & disposed of as VLDL - ↓LPL - Glucagon Secretion Deficiency Treatment: - Insulin (Always Necessary)

An 8-year-old boy with a confirmed history of insulin-dependent diabetes (type I) was brought to the emergency department complaining of dizziness and rapid heart rate. Blood is taken and the following results have been obtained: Blood Glucose (mg/100ml): 40 (Normal: 70-100) Blood pH: 7.3 (Normal: 7.3-7.5) Blood Ketone Bodies (mg/100ml): 2 (Normal: 0-3) Which of the following would be expected at the time of his admission to the hospital? Decreased levels of circulating hemoglobin A1c Increased lipolysis in adipose tissue Increased levels of circulating epinephrine and glucagon Decreased entry of glucose into adipose cells Decreased levels of circulating insulin

hyperglycemia An increased concentration of glucose in the blood leads to an increase in nonenzymatic glycosylation of hemoglobin, which results in the formation of Hb A1c. This is seen in diabetics, and the concentration of Hb A1c is used as a measure of diabetic control.

An increase in the level of Hb A1c in red blood cells is primarily caused by: anoxia methemoglobinemia hyperglycemia decreased lifetime of the red blood cells increased levels of 2,3-bisphosphoglycerate

D. The patient was diagnosed with maturity-onset diabetes of the young (MODY) caused by this mutation. In glucokinase, binding of glucose normally causes a huge conformational change in the actin fold that creates the binding site for ATP. Although proline and leucine are both nonpolar amino acids, B is incorrect—proline creates kinks in helices and thus would be expected to disturb the large conformational change required (see Chapter 7). In general, binding of the first substrate to an enzyme creates conformational changes that increases the binding of the second substrate or brings functional groups into position for further steps in the reaction. Thus, a mutation need not be in the active site to impair the reaction, and A is incorrect. It would probably take more energy to fold the enzyme into the form required for the transition-state complex, and fewer molecules would acquire the energy necessary (thus, C is incorrect). The active site lacks the functional groups required for an alternate mechanism (thus, E is incorrect).

An individual had a congenital mutation in glucokinase in which a proline was substituted for a leucine on a surface helix far from the active site but within the hinge region of the actin fold. This mutation would be expected to have which one of the following effects? A. It would have no effect on the rate of the reaction because it is not in the active site. B. It would have no effect on the rate of the reaction because proline and leucine are both nonpolar amino acids. C. It would have no effect on the number of substrate molecules reaching the transition state. D. It would probably affect the binding of ATP or a subsequent step in the reaction sequence. E. It would probably cause the reaction to proceed through an alternate mechanism.

to exhibit clinical symptoms of a type II (NIDDM) diabetes In liver and beta-cells, lack of GLUT-2 will reduce the rate of glucose intake. Consequently, the induction of insulin secretion by beta-cells will be severely damaged. Other tissues listed depend on other glucose transport proteins.

An individual was found to have a homozygous deficiency in the GLUT-2 gene, resulting in the lack of a normal glucose-2 transporter. Consequently, this individual is expected: to exhibit clinical symptoms of a type II (NIDDM) diabetes to have lost the ability to accelerate glucose uptake by muscle cells when exposed to insulin to have an impaired ability of glucose uptake by brain tissue to exhibit a normal ability for facilitating glucose transport into liver cells to lack the ability of carrying out lipogenesis from glucose in adipose tissue

Correct answer = D. Many individuals with type 2 diabetes are obese, and almost all show some improvement in blood glucose with weight reduction. Symptoms usually develop gradually. These patients have elevated insulin levels and usually do not require insulin (certainly not 6 hours after a meal) until late in the disease. Glucagon levels are typically normal.

An obese individual with type 2 diabetes typically: A. benefits from receiving insulin about 6 hours after a meal. B. has a lower plasma level of glucagon than does a normal individual. C. has a lower plasma level of insulin than does a normal individual early in the disease process. D. shows improvement in glucose tolerance if body weight is reduced. E. shows sudden onset of symptoms.

B. Insulin release is dependent on an increase in the ATP/ADP ratio within the pancreatic β-cell. In MODY, the mutation in glucokinase results in a less active glucokinase at glucose concentrations that normally stimulate insulin release. Thus, higher concentrations of glucose are required to stimulate glycolysis and the TCA cycle to effectively raise the ratio of ATP to ADP. Answer A is incorrect because cAMP levels are not related to the mechanism of insulin release. Answer C is incorrect because initially transcription is not involved because insulin release is caused by exocytosis of preformed insulin in secretory vesicles. Answer D is incorrect because the pancreas will not degrade glycogen under conditions of high blood glucose, and answer E is incorrect because lactate does not play a role in stimulating insulin release.

Assume that an increase in blood glucose concentration from 5 to 10 mM would result in insulin release by the pancreas. A mutation in pancreatic glucokinase can lead to MODY because of which one of the following within the pancreatic β-cell? A. A reduced ability to raise cAMP levels B. A reduced ability to raise ATP levels C. A reduced ability to stimulate gene transcription D. A reduced ability to activate glycogen degradation E. A reduced ability to raise intracellular lactate levels

F. In the absence of insulin, glucagon-stimulated activities predominate. This leads to the activation of PKA, the phosphorylation and inactivation of glycogen synthase, the phosphorylation and activation of phosphorylase kinase, and the phosphorylation and activation of glycogen phosphorylase.

Consider a person with type 1 diabetes who has neglected to take insulin for the past 72 hours and also has not eaten much. Which one of the following best describes the activity level of hepatic enzymes involved in glycogen metabolism under these conditions?

A. People with type 1 diabetes exhibit high blood glucose levels caused by the lack of insulin and the inability of peripheral tissues to effectively transport glucose from the blood into the tissue. Individuals on diets would have low blood glucose levels because the majority of energy is being derived from fat and ketone body production. The level of free fatty acids in the blood would be elevated under both conditions owing to the high glucagon-to-insulin ratio. Lactate levels are low under both conditions because glycolysis does not need to act rapidly to provide energy. Six- and eight-carbon dicarboxylic acids in the serum are indicative of a problem in oxidizing fatty acids (MCAD deficiency), which does not apply under these conditions. Carnitine levels in the blood would be expected to be low in both individuals because the tissues require the carnitine for fatty acid oxidation, which is a primary energy source for each type of individual.

Elevated levels of ketone bodies can be found in the blood of people with untreated type 1 diabetes and individuals on severe diets. A major difference in the laboratory findings of metabolites in the blood of each type of individual (type 1 diabetes vs. the diet) would be which of the following? A. Glucose levels B. Free fatty acid levels C. Lactate levels D. Six- and eight-carbon dicarboxylic acid levels E. Carnitine levels

Reduced insulin release after eating. A primary driver of insulin release, in addition to elevated blood glucose levels, is the action of the incretins on the pancreas, further stimulating the pancreas to release insulin. Glucose given intravenously to individuals, bypassing the digestive process, results in a reduced secretion of insulin as compared to a similar rise in glucose levels through normal digestive means. The incretins include glucagon-like peptde-1 (GLP-1) and gastric inhibitory polypeptide (GIP). Glucagon release will raise blood glucose levels, not reduce them, and growth hormone release is not affected by gastric bypass surgery.

Gastric bypass surgery performed on an individual who was obese, and developed type 2 diabetes, dramatically alleviated the hyperglycemia due to the diabetes than would be expected for their weight loss alone. This might occur due to which one of the following? Reduced insulin release after eating. Reduced glucagon release after eating. Increased glucagon release after eating. Increased growth hormone release after eating. Increased insulin release after eating.

Lipoprotein lipase levels are very low in diabetes type 1; In diabetes type II the amounts of TAGS in VLDL is very large combined with less than optimal levels of lipoprotein lipase

In diabetes type I and type II there are increased levels of VLDL in the patient's blood. However, there are two very different reasons for the increased VLDL in these two types of diabetes. Please explain the mechanism for increased VLDL levels in diabetes type I (insulin dependent) and in diabetes type II (insulin independent). Lipoprotein lipase levels are very low in diabetes type II; In diabetes type I the amounts of TAGS in VLDL is very large combined with less than optimal levels of lipoprotein lipase Apo CII levels are very low in diabetes type I; In diabetes type II the amounts of TAGS in VLDL is very large combined with less than optimal levels of lipoprotein lipase Lipoprotein lipase levels are very low in diabetes type 1; In diabetes type II the amounts of TAGS in VLDL is very large combined with less than optimal levels of lipoprotein lipase Lipoprotein lipase levels are very low in diabetes type I; In diabetes type II the amounts of TAGS in VLDL is very large combined with less than optimal levels of apo CII

a lower insulin level, a higher blood glucose level, and a higher blood fatty acid level Insulin levels are very low or nonexistent in untreated type 1 diabetes, which permits blood glucose levels to increase. The inhibitory effect of glucose on glucagon excretion also requires insulin. Thus, at least initially, the glucagon levels increase despite the high glucose levels, which further increases glucose levels, activates hepatic adenylyl cyclase, and increases fatty acid mobilization from the adipocytes. These actions increase circulating fatty acid levels and provide more substrate for hepatic ketone body formation, resulting in a ketoacidosis that is more severe than that observed during fasting. In the normal physiologic response to fasting, the regulatory mechanisms are intact.

Ketoacidosis is both a normal physiologic response to prolonged caloric restriction (e.g., during fasting or stringent dieting) and an abnormal process associated with type 1 diabetes. Which one of the following sets of events best distinguishes the untreated diabetic process from the physiologic one of fasting? Relative to the fasting state, in type 1 diabetes, there is: a lower insulin level, a higher blood glucose level, and a higher blood fatty acid level a lower insulin level, a higher blood glucose level, and a lower blood fatty acid level an increased glucagon/insulin ratio, a higher hepatocyte cAMP level, and an equivalent blood glucose level a lower insulin level, a higher blood glucose level, and an equivalent blood fatty acid level an equivalent insulin level, a higher hepatocyte cAMP level, and a higher blood glucose level

D. Metformin interrupts the Cori cycle (gluconeogenesis in the liver using lactate, derived from the muscle, as a source of carbons). The heart, with its huge mitochondrial content and oxidative capacity, uses lactate as fuel and easily metabolizes the excess lactate (which is why heart failure is a contraindication to the use of metformin; otherwise, lactic acidosis would occur). The red blood cells, renal medullary cells, and tissues of the eye all use anaerobic glycolysis to generate energy, producing lactate, but cannot use lactate as a fuel.

Metformin is a medication used in treating diabetes mellitus type 2. One of its actions is to decrease hepatic gluconeogenesis. A theoretical concern with this medication was lactic acidosis, which in practice does not occur in patients taking metformin. Which one of the following explains why lactic acidosis does not occur with the use of this medication? A. The Cori cycle overcomes the lactate buildup in the liver. B. Red blood cells use lactate as fuel. C. Renal medullary cells use lactate as fuel. D. The heart uses lactate as fuel. E. The eye uses lactate as fuel.

A. Channels which open in response to a change in ion concentration across the membrane (which results in a change in membrane potential, or voltage, across the membrane) are known as voltage-gated channels. The calcium influx is not passive diffusion because a carrier is required (the channel). This is not an active transport process because calcium is flowing down its concentration gradient and the cell is not concentrating calcium within it. A ligand-gated channel opens when a particular ligand binds to it, not when the membrane potential changes. There is no phosphorylation event required in the opening of this calcium channel, so it is not an example of a phosphorylation-gated channel.

One manner in which type 2 diabetes occurs is via a reduction in the release of insulin from the pancreas. The release of insulin from the β-cells of the pancreas requires Ca2+ influx through a channel that is activated by a change in the membrane potential across the plasma membrane. The movement of calcium across the membrane is an example of which one of the following? A. Voltage-gated channel B. Passive diffusion C. Active transport D. Ligand-gated channel E. Phosphorylation-gated channel

Hemoglobin

Patients with poorly controlled diabetes have elevated levels of blood glucose. One severe consequence of the hyperglycemia is an increase in glucose attachment to serum proteins. Which of the following proteins, when glycosylated, is an excellent measure of the length of time someone has suffered from an episode of hyperglycemia? Hemoglobin Albumin Cholesterol Fatty acids Transferrin

Correct answer = D. Low insulin levels favor the liver producing ketone bodies, using acetyl coenzyme A generated by β-oxidation of the fatty acids provided by hormone-sensitive lipase (HSL) in adipose tissue (not liver). Low insulin also causes activation of HSL, decreased glycogen synthesis, and increased gluconeogenesis and glycogenolysis. A. Increased HSL B. Increased Gluconeogenesis C. Increased Glycogenolysis E. Decreased Glycogenesis

Relative or absolute lack of insulin in humans would result in which one of the following reactions in the liver? A. Decreased activity of hormone-sensitive lipase B. Decreased gluconeogenesis from lactate C. Decreased glycogenolysis D. Increased formation of 3-hydroxybutyrate E. Increased glycogenesis

C: ATP and NADPH. The β-cells of the pancreas monitor both ATP and NADPH levels in order for insulin release to occur. The NADPH levels are increased through enhanced pyruvate cycling (see the figure below), which occurs when pyruvate levels increase (which is correlated with an increase in glu- cose levels within the β-cell). Increased glucose also leads to an increase in ATP, which leads to changes in ion fluxes across the membrane, resulting in insu- lin release. Glucose-6-phosphate, carbon dioxide, and NADH are not necessary for insulin release in response to glucose.

Sequelae of insulin resistance in type 2 diabetes mellitus and metabolic syndrome is reduced secretion of insulin in response to increases in blood glucose. Insulin release from the pancreas appears to be dependent upon increase in concentration of which pair of metabolites? (A) ATP and CO2 (B) ATP and NADH (C) ATP and NADPH (D) Glucose-6-phosphate and CO2 (E) Glucose-6-phosphate and NADH

TZDs via gene expression regulation make the adipocytes more able to retain TAGs. This results in more insulin sensitive in the tissues since there is fewer fatty acids in circulation and less ecto-deposition in the muscle and the liver. However, the patient continues to gain weight. TZD as agonist of PPAR gamma regulate gene expression in several tissues but of particular importance is the effect in adipocytes. Since free fatty acids are the ones causing the ecto deposition and insulin resistance in tissues lowering the levels in blood by retaining them better in the adipocytes helps the patient to control better the hyperglycemia.

The drug Avandia (rosiglitazone a TZD type of drug) is effective in lowering blood glucose in patients with type 2 diabetes, but also seems to carry an increased risk of heart attack. What is the mechanism of action of TZDs such as Avandia to lower glucose levels in the blood? TZDs via gene expression regulation make the adipocytes more able to retain TAGs. This results in more insulin sensitive in the tissues since there is fewer fatty acids in circulation and less ecto-deposition in the muscle and the liver. However, the patient continues to gain weight. TZDs via gene expression regulation make the adipocytes more able to retain TAGs. This results in more insulin sensitivity in the tissues since there is less fatty acids in circulation and less ecto deposition in the muscle and the liver. However, the patient continue to lose weight. TZDs via gene expression regulation make the adipocytes less able to retain TAGs. This results in more insulin sensitivity in the tissues since there is less fatty acids in circulation and less ecto deposition in the muscle and the liver. However, the patient continue to lose weight. TZDs receptor mediated signal transduction make the adipocytes more able to retain TAGs. This results in more insulin insensitivity in the tissues since there are more fatty acids in circulation and more ecto deposition in the muscle and the liver. However, the patient continue to gain weight.

Type I diabetes Poorly controlled diabetes, with its resultant elevated blood glucose levels, can result in the overproduction of sorbitol and its accumulation in specific tissues, with long-term pathologic consequences.

The formation of cataracts, peripheral neuropathy, and/or vascular disease has been, under specific circumstances, linked to the accumulation of sorbitol in the involved tissues. Which ONE of the following conditions can contribute to the accumulation of sorbitol? Mutation in the GLUT-2 sugar transporter Lactose intolerance Sucrase deficiency Overconsumption of fructose Type I diabetes

B: Ability to produce insulin. By defini- tion, a type 1 diabetic cannot produce insulin. A type 2 diabetic produces insulin, but has become resistant to the effects of insulin. Weight does not differentiate between type 1 and 2 diabetics (although most type 2 diabetics are overweight). Blood glucose levels are elevated in both types of diabetes and cannot differentiate between them. Neither LDL levels nor triglyceride levels will differentiate between these two major forms of diabete

The major, defining difference between a type 1 diabetic and a type 2 diabetic is which of the following? (A) Weight (B) Ability to produce insulin (C) LDL levels (D) Blood glucose levels (E) Serum triglyceride levels

A. The patient's enzyme has a lower Km than the normal enzyme and therefore requires a lower glucose concentration to reach ½Vmax. Thus, the mutation may have increased the affinity of the enzyme for glucose, but it has greatly decreased the subsequent steps of the reaction leading to formation of the transition-state complex, and thus, Vmax is much slower. The difference in Vmax is so great that the patient's enzyme is much slower whether you are above or below its Km for glucose. You can test this by substituting 2 mM glucose and 4 mM glucose into the Michaelis-Menten equation, v = Vmax S/(Km + S) for the patient's enzyme and for the normal enzyme. The values are 0.0095 and 0.0129 for the patient's enzyme versus 23.2 and 37.2 for the normal enzyme, respectively (thus, B and C are incorrect). At near-saturating glucose concentrations, both enzymes will be near Vmax, which is equal to kcat times the enzyme concentration. Thus, it will take nearly 500 times as much of the patient's enzyme to achieve the normal rate (93 ÷ 0.2), and so C is incorrect. E is incorrect because rates change most as you decrease substrate concentration below the Km. Thus, the enzyme with the highest Km will show the largest changes in rate.

The pancreatic glucokinase of a patient with MODY had a mutation replacing a leucine with a proline. The result was that the Km for glucose was decreased from a normal value of 6 mM to a value of 2.2 mM, and the Vmax was changed from 93 U/mg protein to 0.2 U/mg protein. Which one of the following best describes the patient's glucokinase compared with the normal enzyme? A. The patient's enzyme requires a lower concentration of glucose to reach ½Vmax. B. The patient's enzyme is faster than the normal enzyme at concentrations of glucose <2.2 mM. C. The patient's enzyme is faster than the normal enzyme at concentrations of glucose >2.2 mM. D. At near-saturating glucose concentration, the patient would need 90 to 100 times more enzyme than normal to achieve normal rates of glucose phosphorylation. E. As blood glucose levels increase after a meal from a fasting value of 5 to 10 mM, the rate of the patient's enzyme will increase more than the rate of the normal enzyme.

B: Cortisol stimulation of amino acid release from the muscle. The weight loss seen in type 1 diabetics (untreated) results from the need of the liver for gluconeogenic precursors, many of which are derived from amino acids obtained from muscle protein breakdown. Cortisol release will signal the muscle to release amino acids for use by the liver. Glucagon sig- nals triglyceride degradation, not production. Insulin does signal an inhibition of fatty acid oxidation, but that is not occurring in an individual who does not make insulin (type 1 diabetic). Since the muscle is oxidizing fatty acids for energy, there is no activation of the AMP-activated protein kinase, as the energy levels are not low. Muscle acetyl-CoA carboxylase 2 (which produces malonyl-CoA, which would inhibit fatty acid oxidation via inhibiting carnitine palmitoyl transferase 1) is not activated under these conditions, due to the lack of insulin.

The polyphagia observed in the untreated type 1 diabetic, who has lost 6 lb in the last 2 weeks, is due to which of the following? (A) Glucagon stimulation of triacylglycerol production (B) Cortisol stimulation of amino acid release from the muscle (C) Insulin-induced inhibition of fatty acid oxidation (D) AMP kinase-induced activation of GLUT4 transporters (E) Activation of muscle acetyl-CoA carboxylase-2

C. Patient #3 Patient #2 has a normal fasting blood glucose (FBG) but an impaired glucose tolerance (GT) as reflected in her blood glucose level at 2 hours and, so, is described as prediabetic Patient #1 has a normal FBG and GT, whereas patient #3 has diabetes.

Three patients being evaluated for gestational diabetes are given an oral glucose tolerance test. Based on the data shown below, which patient is diabetic? A. Patient #1 B. Patient #2 C. Patient #3 D. None

B. Patient #2 Patient #2 has a normal fasting blood glucose (FBG) but an impaired glucose tolerance (GT) as reflected in her blood glucose level at 2 hours and, so, is described as prediabetic Patient #1 has a normal FBG and GT, whereas patient #3 has diabetes.

Three patients being evaluated for gestational diabetes are given an oral glucose tolerance test. Based on the data shown below, which patient is prediabetic? A. Patient #1 B. Patient #2 C. Patient #3 D. None

D: Osmotic imbalance due to increased glucose levels in the urine. In an untreated type 1 diabetic, glucose levels in the blood exceed the renal threshold for reabsorption of the glucose from the urine, so blood glucose levels rise in the urine. This creates an osmotic imbalance, which forces more water into the urine, leading to polyuria (frequent urination). This is not due to urea production (and, since a type 1 diabetic does not produce insulin, insulin cannot be stimulating urea production). It is not due to an increase of ketones in the blood. And, since insulin is not present, it cannot be due to insulin stimulation of glucose resorption in the kidney.

Type 1 diabetics, prior to diagnosis, display polydipsia, polyuria, and polyphagia. The polyuria is due to which of the following? (A) Insulin stimulation of urea production (B) Osmotic imbalance due to elevated ketones in the blood (C) Osmotic imbalance due to reduced glucose levels in the urine (D) Osmotic imbalance due to increased glucose levels in the urine (E) Insulin stimulation of glucose resorption in the kidney

Hyperinsulinemia Insulin is considered a fetal growth factor because babies born to women with diabetes mellitus and hyperinsulinemia are larger with more adipose tissue deposition. Babies with congenital hyperinsulinemia are also born larger. Type 2 Diabetes: - Insulin Resistance & β Cells cannot produce enough Insulin - Age of Onset: Mostly after 35 - 90% of Diagnosed Diabetes - Very Strong Genetic Predisposition - ↑Insulin Secretion → 2nd Mutation → ↓Glucose Stimulated Secretion → ↓Arginine Stimulated Secretion → ↓β Cells Pathophysiology: - Insulin Resistance & β Cells cannot produce enough Insulin - Abnormal Fat & Muscle Metabolism - Hyperosmolar State - Dyslipidemia - Typically Obese (↑Leptin, ↑TNFα, ↑IL-6, ↓Adiponectin) - Ectopic Lipid Deposition & Nutrient Excess → (+)mTOR → Mitochondrial Dysfunction Affects only PI₃K Pathway: - Mostly Post-Receptor Dysfunction - Ser/Thr Phosphorylation (Instead of Tyr Phosphorylation) Impaired Glucose Tolerance (IG): - due to ↓Peripheral utilization of glucose Impaired Fasting Glucose (IFG): - ↑Hepatic Glucose Produced (Hyperglycemia) Treatment: - Diet, Exercise, Hypoglycemic Drugs, May or may not need Insulin - Metformin (Targets AMPK, activated) →↑Glucose Uptake by Muscle & ↓Glucose production by Liver - Thiazolidinediones (Targets PARγ) →↑Insulin Action in Liver & Muscle

Which ONE of the following best describes a characteristic of newly diagnosed type 2 diabetes mellitus not usually present in type 1 diabetes mellitus? Increased lipolysis in adipose Insulin required as therapy Hyperinsulinemia Hyperglycemia Ketosis

Hyperglycemia Hyperglycemia is caused by increased hepatic production of glucose, combined with diminished use of glucose by muscle and adipose tissues. Ketonemia is usually minimal or absent in patients with Type 2 Diabetes (T2D) because the presence of insulin, even in the presence of insulin resistance, restrains hepatic ketogenesis.

Which ONE of the following changes is unique to acute type I diabetes mellitus and is not present in starvation? Increased protein synthesis Increased ketogenesis in liver Hyperglycemia Increased lipolysis in adipose Insulin blood less than observed in normal absorptive period

Concentration of insulin in plasma Type 1 patients will have no circulating insulin, whereas type 2 patients will have normal or even elevated insulin.

Which ONE of the following could best be used to differentiate between type 1 (insulin-dependent) diabetes and type 2 (non-insulin-dependent) diabetes in a recently diagnosed, untreated patient? Concentration of blood glucose Age of onset of disease Presence of diabetic neuropathy Concentration of insulin in plasma Level of Hb A1c in red blood cells

Insulin is present in blood Patients with type 2 diabetes (formerly called noninsulin-dependent diabetes) have a combination of insulin resistance and dysfunctional β cells, but do not require insulin to sustain life.

Which ONE of the following is characteristic of type 2 diabetes? Ketosis is commonly observed Insulin is present in blood Onset typically occurs during childhood or puberty Accounts for approximately 10% of all diabetics Always requires treatment with insulin

Exercise Exercise promotes glucose uptake into muscle and decreases the need for exogenous insulin. Patients are advised, therefore, to check blood glucose levels before or after intensive exercise to prevent or abort hypoglycemia. Type 1 Diabetes: - Destroyed β Cells - Age of Onset: Childhood or Puberty - Undernourished Pathophysiology: - Enlarged Peripancreatic Lymph Nodes - Cytokines (TNFα, IFNγ, IL-1) Diabetic Ketoacidosis: - ↓Insulin →↑FA Ox →↑↑↑Acetyl CoA →↑Ketones - ↑HSL (AT→FFAs) - Nausea & Vomiting - Abdominal Pain (Pancreatitis due to ↑↑↑TGs) - Kussmaul's breathing (↓HCO₃, Hyperkalemic Metabolic Acidosis) - Fruity Odor (Acetone) - Hyperglycemia - Dehydration (Glucosuria w/ Water Loss) - Hypotension & Coma - TX: Dextrose & Normal Saline (DNS) + Insulin Hyperglycemia: - ↑Glycogenolysis & ↑Gluconeogenesis - ↑A1c Hypertriglyceridemia: - ↑↑↑FFAs packed & disposed of as VLDL - ↓LPL - Glucagon Secretion Deficiency Treatment: - Insulin (Always Necessary)

Which ONE of the following will decrease the need for insulin in an insulin-dependent diabetic (type 1)? Consumption of pancakes with maple syrup Injection of glucagon Exercise Injection of epinephrine Ingestion of Diet Pepsi

A. Upon insulin release, the cAMP phosphodiesterase is activated, reducing cAMP levels in the liver, thereby leading to inactivation of protein kinase A. In addition, protein phosphatase 1 has become active and dephosphorylates the enzymes that were phosphorylated by protein kinase A. Therefore, PFK-2 is not phosphorylated, which leads to an active kinase activity and a inactive phosphatase activity (choices A, D, or E). The active kinase of PFK-2 produces more fructose-2,6-bis- phosphate, leading to the activation of PFK-1 (answers A through D; combined with PFK-2 activity, now only choice A or D can be correct). Insulin stimulates pre- formed GLUT4 transporters in the muscle to fuse with the plasma membrane, thereby enhancing glucose transport into the muscle (choices A through C; combined with the other two columns, only choice A can be correct)

Which of the following changes in enzyme activity will occur within 1 h of a type 1 diabetic taking an injection of insulin?

D. In type 1 diabetes mellitus, the patient is producing low or no insulin, which means that C-peptide levels are very low or nondetectable. The patients are usually thin because low insulin does not promote adipose tissue storage of triacylglycerol. There is no insulin resistance in type 1 because the patients are not producing insulin. Insulin resistance at the cellular level primarily causes type 2 diabetes mellitus. Because of the resistance, insulin levels are generally elevated to attempt to overcome this resistance. With elevated insulin levels, adipose tissue storage of triacylglycerol is high and the patient is almost always overweight. There are exceptions to this general statement, and those are patients who have inherited a mutation leading to type 2 diabetes, such as maturity-onset diabetes of the young (MODY).

Which one of the following is a major difference between patients with diabetes mellitus types 1 and 2? A. In type 1, insulin levels are very low and insulin resistance is high. B. In type 2, insulin levels are very low and insulin resistance is high. C. In both types, the patient is usually at or below ideal body weight. D. In type 1, C-peptide levels are very low. E. In type 1, the patient is usually overweight.

Correct answer = A. Elevated blood glucose occurs in type 1 diabetes (T1D) as a result of a lack of insulin. In type 2 diabetes (T2D), hyperglycemia is due to a defect in β-cell function and insulin resistance. The hyperglycemia results in elevated hemoglobin A1c levels. Ketoacidosis is rare in T2D, whereas obesity is rare in T1D. C (connecting)-peptide is a measure of insulin synthesis. It would be virtually absent in T1D and initially increased then decreased in T2D. Both forms of the disease show complex genetics.

Which one of the following is characteristic of untreated diabetes regardless of the type? A. Hyperglycemia B. Ketoacidosis C. Low levels of hemoglobin A1c D. Normal levels of C-peptide E. Obesity F. Simple inheritance pattern

B: C-peptide levels. It is difficult, at times, to differentiate type 1 diabetes mellitus and ketoacidosis from type 2 diabetes mellitus hyperosmo- lar state and lactic acidosis unless testing for acetate and/or β-hydroxybutyrate (ketone bodies) are specifically ordered when acidosis is noticed. At this point in the patient's disease process, ketone bodies should be nor- mal. The patient is already on insulin, so insulin levels would not be helpful. Measurement of blood glucose levels, whether fasting or not, and determination of Hb1AC levels cannot differentiate type 1 from type 2 diabetes (since the patient is on insulin). Since insulin is secreted as a macromolecule and does not become active until C-peptide is cleaved from the macromolecule, a C-peptide level would be helpful in this differentiation. Type 1 diabetes mellitus (no endogenous insulin produced) should give a very low or nonexistent level of C-peptide, whereas type 2 diabetes mellitus (insu- lin resistance) should give a normal or high level of C-peptide. Exogenous (commercial) insulin lacks the C-peptide, so the injected insulin will not interfere with this measurement.

You see a 56-year-old female patient in follow-up after discharge from the hospital. She was treated for ketoacidosis and hyperglycemia and now is on basal and rapid acting insulins. You wonder if she really has type 1 diabetes mellitus and was in ketoacidosis or has type 2 diabetes mellitus and had a hyperosmolar state with lactic acidosis. Which of the following lab tests would help you determine whether this patient has type 1 or type 2 diabetes mellitus? (A) Insulin levels (B) C-peptide levels (C) Fasting blood glucose levels (D) Random blood glucose levels (E) Hemoglobin A1C levels

B: Self injection of insulin. This patient could inject exogenous insulin to simulate an insuli- noma. The symptoms and lab findings would be iden- tical unless a C-peptide analysis was done. Injecting insulin between meals leads to hypoglycemia as the insulin stimulates glucose transport from the blood into the peripheral tissues, in the absence of dietary glucose. The figure below compares the effects of hypoglycemia (what is occurring in this case) versus hyperglycemia (as in an untreated diabetic) on a patient. Injecting glu- cagon would cause release of glucose from glycogenoly- sis (and gluconeogenesis), resulting in a higher blood glucose level. Amylin is a compound which blocks the action of glucagon, so an amylin blocker would be the same as injecting glucagon (blocking amylin activity would increase glucagon activity, since amylin is no longer active). Carbohydrate loading is an attempt to raise glycogen stores for more glucose availability dur- ing prolonged exercise and would not lead to hypogly- cemic episodes. Metformin blocks liver gluconeogenesis during the fasting state, so more fatty acids are utilized. It also reduces insulin resistance. It does not stimulate insulin release and does not produce hypoglycemia

Your 20-year-old male patient has had multiple episodes of lightheadedness, sweating, fatigue, tremor, and intense hunger. He had one seizure. During two of these episodes, his blood glucose was 40 mg/dL. This patient was desperately trying to get a discharge from the military, and you suspected he was inducing his symptoms by doing which of the following? (A) Self injection of glucagon (B) Self injection of insulin (C) "Carb loading" before exercise (D) Taking metformin before exercise (E) Taking an amylin blocker

D: It inhibits glucagon secretion. Pram- lintide is an amylin agonist used to lower postprandial blood glucose. Amylin is a peptide hormone secreted by the beta cells of the pancreas (with insulin), and inhibits glucagon secretion when blood glucose levels are elevated after a meal (thus aiding insulin action). Glucagon stimu- lates release of glucose from glycogen and further raises blood glucose. Insulin stimulates glycogenesis and storage of glucose which lowers blood glucose. Inhibiting insulin secretion would worsen the problem of high blood glu- cose levels. Decreasing glucose-6-phosphate or stimulat- ing hexokinase or glycogen phosphorylase would increase glycogenolysis and raise blood glucose, which is opposite what one wants to accomplish in a diabetic patient.

Your diabetic patient has recently been placed on pramlintide (Symlin) to help control his diabetes. Which of the following best describes the mechanism of action of this medication? (A) It decreases glucose-6-phosphate (B) It increases hexokinase (C) It stimulates glycogen phosphorylase (D) It inhibits glucagon secretion (E) It inhibits insulin secretion

B: Metformin blocks hepatic gluconeogenesis. Metformin leads to a reduction of hepatic gluconeogenesis. This is accomplished through the activation of the AMP-activated protein kinase, which phosphorylates and sequesters within the cytoplasm TORC2, which is a coactivator of CREB activity (a transcription factor needed for expression of two glu- coneogenic enzymes, PEP carboxykinase and glucose- 6-phosphatase). Thus, when TORC2 is absent from the nucleus, gluconeogenesis is impaired as the synthesis of two key enzymes is greatly reduced. One of the major gluconeogenic precursors is lactate, generated from the red blood cells and exercising muscle. In the Cori cycle, two lactates are converted to one glucose, which is then exported. If gluconeogenesis is blocked, lactate is not utilized and its levels can increase, and poten- tially lead to lactic acidosis. However, in the absence of congestive heart failure or renal insufficiency, this does not occur. The heart, with its massive amount of mus- cle and mitochondria, can utilize the lactate for energy unless the heart is dysfunctional or has lost muscle mass. Good, functional kidneys can also overcome the lactate imbalance caused by metformin treatment. Metformin does decrease the insulin resistance, but this does not increase lactate in the aerobic state. Metformin does not inhibit the TCA cycle, glycolysis, or dietary protein absorption.

Your obese patient has type 2 diabetes mellitus and you have started him on metformin. One of the possible complications of metformin therapy is lactic acidosis. Why is this a concern with metformin therapy? (A) Metformin reduces insulin resistance (B) Metformin blocks hepatic gluconeogenesis (C) Metformin blocks the TCA cycle (D) Metformin inhibits glycolysis (E) Metformin inhibits dietary protein absorption

B: PPAR-γ Thiazolidinediones (TZDs), of which pioglitazone is a member bind to peroxi- some proliferator activated receptor-γ (PPAR-γ) in the adipocyte and activate the synthesis and release of adi- ponectin, which acts on target cells to reduce blood glucose levels (by upregulating GLUT4 content of the membranes) and to reduce circulating triglyceride levels (through phosphorylation and inhibition of acetyl-CoA carboxylase 2, which relieves the inhibition of carni- tine palmitoyl transferase I). While adiponectin levels rise, which leads to a stimulation of the AMP-activated protein kinase, neither of those effects is due to a direct interaction with the TZD. LKB1, an upstream kinase responsible for activating the AMP-activated protein kinase, and leptin are not involved in the response to TZDs. The structure of Actos (pioglitazone) is shown below

Your patient with metabolic syndrome is in for a checkup. His HbA1C is 9.0 and his fasting triglycerides are 325 mg/dL. You prescribe pioglitazone (Actos) to better treat his diabetes, but nothing else specific for the high lipids. A month later, the fasting triglyceride levels have dropped to 155 mg/dL due to a direct activation of which of the following? (A) AMP-activated protein kinase (B) PPAR-γ (C) Leptin (D) Adiponectin (E) LKB1

B: Potato. Amounts of simple carbo- hydrates in a meal are the most reliable indicators of postprandial rise in blood glucose. As the HbA1C nears normal or target values, fasting blood glucose values are usually normal and postprandial glucose values have a much more important effect on the HbA1C. Proteins, fats, and complex carbohydrates are absorbed more slowly than simple carbohydrates. This can be viewed as the glycemic index (the ability of a food to rapidly raise blood glucose). A higher number for the glycemic index means a more rapid and higher rise of blood glu- cose. Potatoes have the highest glycemic index of the mentioned foods and are composed of simple carbohy- drates. Broccoli has more complex carbohydrates and a much lower glycemic index. Meat is mostly proteins and fats. Milk contains proteins and so has a lower gly- cemic index than potatoes (whole milk also has fats and an even lower glycemic index). Diet drinks contain no carbohydrates or calories.

Your patient with type 2 diabetes mellitus is usually in good control with an HbA1C of 7.1 and fasting blood glucose values between 90 and 100 mg/dL. His problem is with his 1-h postprandial glucose levels at lunch and dinner. A recall of his usual diet reveals some type of meat, potato, broccoli, milk, and diet drink at these meals. Which of these foods is most likely responsible for his postprandial high blood glucose? (A) Meat (B) Potato (C) Broccoli (D) Milk (E) Diet drink

B: Humulin R is complexed with zinc, which slows its absorption. Humulin R (regular acting) is a hexamer complexed with zinc. After injection, the concentration of the insulin has to be reduced (through diffusion) for monomers and dimers of insulin to leave the zinc complex. This dramatically slows the time of insulin appearing in the circulation. Humalog, with a slightly different amino sequence, is not complexed with zinc and is absorbed much more rapidly from the injection site than Humulin R. Both types of insulin are taken subcutaneously, not orally, even when using an insulin pump. Pumps will not alter absorption, just delivery and time of delivery of the insulin. Humalog is not complexed with manganese.

Your type 1 diabetic patient was managing their disease using a combination of Humulin R and Humalog. The Humalog is more rapid acting than the Humulin R due to which of the following? (A) Humalog is taken orally, rather than subcutaneously (B) Humulin R is complexed with zinc, which slows its absorption (C) Humalog is complexed with manganese, which accelerates its absorption (D) Humulin R is taken orally, which slows its absorption (E) Humalog is taken through an insulin pump mechanism

B: Inhibition of the electron transfer chain. Metformin partially inhibits complex I of the electron transport chain. This leads to reduced ATP pro- duction, which, as energy is required, rapidly increases AMP levels due to the adenylate kinase reaction. The increase in AMP levels leads to the activation of the AMP-activated protein kinase (AMPK), which is the pri- mary messenger for metformin's effects. Metformin does not activate adenylate cyclase, nor does LKB1 (it has been postulated that LBK1 is constantly phosphorylat- ing the AMP-activated protein kinase, but a phosphatase is always inactivating the AMPK. When AMP levels rise, however, AMP inhibits the phosphatase, leading to fully active AMPK). Metformin also has no direct effect on the rate-limiting step of purine production, amidophos- phoribosyltransferase, or of adenylate kinase.

Your type 2 diabetic patient has been taking metformin for the past 6 months and has reduced fasting blood glucose levels from 185 to 112 mg/dL. This occurs due to which of the following effects of metformin? (A) Activation of adenylate cyclase (B) Inhibition of the electron transfer chain (C) Activation of LKB1 (D) Stimulation of amidophosphoribosyltransferase (E) Stimulation of adenylate kinase


Ensembles d'études connexes

Computer Science 307: Software Engineering : Chapter 1

View Set

Finance Test (Chapters 6, 7, & 8)

View Set

Government Test 3- Review(Part 3)

View Set

renal part 2 (fluid and e- changes) - exam 3

View Set